Course Content
All Previous Years Krok 2 Papers with Explanations
About Lesson

1. Objectively, a 22-year-old patient has numerous non-inflammatory yellowish-brown and pale pink spots on the skin of the chest, neck, shoulders, and the sides of the torso. When the spots are scratched, the sign of “shavings” is observed. The disease onset was 2 years ago, the rashes were first noticed after a summer vacation by the sea. What is the most likely diagnosis in this case?

A. Pityriasis rosea Gibert

B. Erythrasma

C. Streptodermia

D. Tinea versicolor (Pityriasis versicolor) 

E. Syphilitic roseola

Answer: Tinea versicolor (Pityriasis versicolor)


Explanation

The clinical presentation described, including non-inflammatory yellowish-brown and pale pink spots on the skin of the chest, neck, shoulders, and the sides of the torso, along with the observation of “shavings” when the spots are scratched, is indicative of Tinea versicolor (Pityriasis versicolor).

Tinea versicolor is a superficial fungal infection caused by Malassezia species, commonly M. globosa or M. furfur. The fungus interferes with normal skin pigmentation, leading to the development of hypo- or hyperpigmented macules, which are often observed on the trunk. The condition is exacerbated by sweating, and the spots become more noticeable after sun exposure.

The other options can be ruled out based on the clinical features:

A. Pityriasis rosea Gibert: Presents with a herald patch followed by the appearance of smaller scaly lesions, typically on the trunk.

B. Erythrasma: Manifests as well-demarcated brown patches with fine scaling, usually in intertriginous areas.

C. Streptodermia: Generally refers to skin infections caused by Streptococcus bacteria, presenting as erythematous, sometimes vesicular lesions.

E. Syphilitic roseola: A manifestation of secondary syphilis, characterized by a coppery or brown maculopapular rash affecting the trunk and extremities.

In summary, the clinical presentation of non-inflammatory yellowish-brown and pale pink spots on the trunk, along with the observation of “shavings,” aligns with the characteristic features of Tinea versicolor.

2. A 12-year-old girl became acutely ill with a fever of 38.5°C and a rash appearing on her skin. Objectively, her condition is of moderate severity, she has hepatosplenomegaly, fine punctate rash and maculopapular rash on the flexor surfaces of her limbs, lateral surfaces of the trunk, and lower abdomen. Demarcated bluish-pink coloring is observed on her hands and feet. What is the most likely diagnosis in this case? 

A. Measles

B. Scarlet fever

C. Chickenpox

D. Pseudotuberculosis

E. Infectious mononucleosis

Answer: Scarlet fever


Explanation

The clinical presentation described, including an acute onset of fever, hepatosplenomegaly, fine punctate rash, maculopapular rash on the flexor surfaces of the limbs, lateral surfaces of the trunk, and lower abdomen, along with demarcated bluish-pink coloring on the hands and feet, is characteristic of scarlet fever.

Scarlet fever is a streptococcal infection caused by Group A Streptococcus (Streptococcus pyogenes) and is often associated with a preceding streptococcal pharyngitis. The characteristic rash of scarlet fever typically appears on the second day of illness and begins as a fine punctate rash that then becomes more confluent, giving a “sandpaper-like” texture. The rash is most prominent in flexural areas, such as the flexor surfaces of the limbs.

Let’s briefly review the other options and rule them out:

A. Measles: Presents with a prodromal fever, cough, and coryza, followed by the appearance of a maculopapular rash that starts on the face and spreads downward.

C. Chickenpox: Characterized by an itchy vesicular rash that starts on the face, scalp, and trunk and then spreads to other parts of the body.

D. Pseudotuberculosis: Not a common cause of an acute febrile illness with the described rash. Pseudotuberculosis is often associated with Yersinia infections.

E. Infectious mononucleosis: Typically presents with symptoms such as fever, sore throat, and lymphadenopathy. The rash seen in infectious mononucleosis is usually maculopapular but is not as characteristic as the rash seen in scarlet fever.

In summary, the clinical features of an acute febrile illness with hepatosplenomegaly, a fine punctate rash, and a maculopapular rash on flexor surfaces, along with demarcated bluish-pink coloring on hands and feet, align with the diagnosis of scarlet fever.

3. A 25-year-old woman was hospitalized into the maternity hospital at 34 weeks of her pregnancy with complaints of bright-colored bloody discharge with clots that appeared after a defecation. Objectively, the fetal head is palpable near the uterine fundus. Fetal heart rate 140/min. No labor activity. Vaginal examination shows that the cervix is 3 cm long, its opening allows inserting a fingertip, a soft formation can be palpated through the vaginal fornix. The discharge is hemorrhagic and bright-colored. What is the most likely diagnosis in this case?

A. Premature birth 

B. Low-lying placenta 

C. Placenta praevia 

D. Uterine rupture 

E. Placental abruption

Answer: Placenta praevia


Explanation

Placenta praevia is a condition where the placenta implants in the lower uterine segment, partially or completely covering the internal cervical os. This can lead to painless vaginal bleeding, particularly in the third trimester. The bleeding may be sudden and bright-colored, often occurring after activities like defecation or intercourse.

In the scenario described, the 25-year-old woman at 34 weeks of pregnancy presents with bright-colored bloody discharge with clots after defecation. The findings of a soft formation palpable through the vaginal fornix and a cervix that allows inserting a fingertip are indicative of placenta praevia. The fetal head palpable near the uterine fundus suggests that the presenting part is not engaged in the pelvis.

Let’s briefly review the other options and rule them out:

A. Premature birth: The clinical presentation is more suggestive of vaginal bleeding due to placenta-related issues rather than signs of premature birth.

B. Low-lying placenta: While a low-lying placenta can be associated with bleeding, the specific findings described, such as a soft formation through the vaginal fornix, are more characteristic of placenta praevia.

D. Uterine rupture: Uterine rupture is usually associated with severe abdominal pain and signs of fetal distress. The absence of labor activity and the presence of a palpable fetal head make uterine rupture less likely.

E. Placental abruption: Placental abruption typically presents with painful bleeding, and the clinical presentation in this case is more consistent with painless bleeding, characteristic of placenta praevia.

In summary, the clinical features of painless bright-colored bleeding, a soft formation palpable through the vaginal fornix, and a cervix allowing fingertip insertion, along with the absence of labor activity, are indicative of placenta praevia.

4. A 65-year-old patient with a history of arterial hypertension complains of dizziness and palpitations that occurred throughout the last hour. Objectively, the following is observed: blood pressure 80/40 mm Hg, heart rate 150/min., pulse – 106/min. ECG revealed missing P wave and varying RR intervals, ventricular contraction rate is 136-148/min. What aid must be provided to the patient first?

A. Urgent electrical cardioversion 

B. Prescription of 3-blockers intravenously 

C. Pacemaker implantation

D. Prescription of amiodarone intravenously 

E. Prescription of calcium channel blockers intravenously

Answer: Urgent electrical cardioversion


Explanation

The patient’s presentation with dizziness, palpitations, a heart rate of 150/min, blood pressure of 80/40 mm Hg, and an ECG revealing missing P wave and varying RR intervals is indicative of a tachyarrhythmia. Specifically, the absence of P waves and irregular RR intervals suggest atrial fibrillation with a rapid ventricular response.

In the context of severe symptoms and hemodynamic instability, urgent electrical cardioversion is the initial intervention of choice. This procedure aims to restore the normal rhythm by delivering a synchronized electrical shock to the heart. It is especially indicated when the patient is symptomatic and experiences hemodynamic compromise, as in this case with low blood pressure and symptoms of inadequate perfusion.
Let’s briefly review the other options and rule them out:

B. Prescription of β-blockers intravenously: While β-blockers can be used in the management of atrial fibrillation, they are not the first-line treatment in a patient with severe symptoms and hemodynamic instability.
C. Pacemaker implantation: Pacemaker implantation is not the initial intervention for atrial fibrillation with a rapid ventricular response. It may be considered in certain cases, but urgent electrical cardioversion is more appropriate in this acute setting.

D. Prescription of amiodarone intravenously: Amiodarone may be used for rhythm control in stable patients, but in the context of acute symptoms and hemodynamic compromise, urgent electrical cardioversion is more suitable.

E. Prescription of calcium channel blockers intravenously: Calcium channel blockers can be used for rate control in atrial fibrillation, but in a patient with severe symptoms and low blood pressure, urgent electrical cardioversion is the priority to restore sinus rhythm quickly.

In summary, given the acute presentation, hemodynamic instability, and the nature of the rhythm disturbance, urgent electrical cardioversion is the most appropriate initial aid for this patient.

5. A 3-year-old child has episodes accompanied by cyanosis, sudden anxiety, and squatting. Objectively, the child has “drum-stick deformation of the finger phalanges and nails that resemble a clock face. The cardiac dullness boundaries are shifted to the left and right. A systolic tremor can be detected in the second intercostal space near the left edge of the sternum. A coarse systolic murmur can be heard with p.max in the second intercostal space. The second heart sound is weakened over the base of the heart. X-ray shows that the heart is in the form of a “wooden shoe”, the pulmonary pattern is poorly visible. What is the most likely diagnosis in this case? 

A. Dilated cardiomyopathy 

B. Atrial septal defect

C. Ventricular septal defect

D. Primary bacterial endocarditis 

E. Tetralogy of Fallot

Answer: Tetralogy of Fallot


Explanation

The clinical features described, including episodes of cyanosis, sudden anxiety, squatting, “drumstick deformation” of the finger phalanges, and nails resembling a clock face, are indicative of Tetralogy of Fallot (TOF). TOF is a congenital heart defect characterized by four components:

Pulmonary stenosis: This leads to the systolic tremor and a coarse systolic murmur, with the point of maximum intensity (p.max) typically in the second intercostal space near the left edge of the sternum.
Right ventricular hypertrophy: This is responsible for the “drumstick deformation” of the finger phalanges and nails resembling a clock face.

Overriding aorta: The aorta is positioned over both ventricles, allowing oxygen-poor blood from the right ventricle to mix with oxygen-rich blood from the left ventricle.

Ventricular septal defect (VSD): The VSD contributes to the leftward and rightward shift of the cardiac dullness boundaries and the weakened second heart sound over the base of the heart.
The squatting behavior in the child is a compensatory mechanism to increase systemic vascular resistance, which helps in reducing right-to-left shunting of deoxygenated blood and alleviating cyanosis.

Let’s briefly review the other options and rule them out:

A. Dilated cardiomyopathy: Dilated cardiomyopathy is not characterized by the specific findings mentioned in the case, such as pulmonary stenosis, overriding aorta, and ventricular septal defect.

B. Atrial septal defect: Atrial septal defect does not typically present with the “drumstick deformation” of the finger phalanges and nails resembling a clock face.

C. Ventricular septal defect: While VSD is a component of Tetralogy of Fallot, the full spectrum of findings in this case, including pulmonary stenosis, overriding aorta, and specific physical features, points more towards
Tetralogy of Fallot than isolated VSD.

D. Primary bacterial endocarditis: The clinical presentation and physical examination findings are not consistent with bacterial endocarditis.

In summary, the combination of symptoms, physical examination findings, and characteristic X-ray features align with the diagnosis of Tetralogy of Fallot.

6. On the fifth day of life, a newborn presents with excessive abdominal distension with contouring of intestinal loops on the anterior abdominal wall. Meconium passed after the enema. Abdominal X-ray shows dilation of the loops of the large intestine. What is the most likely diagnosis in this case?

A. Portal hypertension 

B. Stenosis of the ileum 

C. Ladd’s syndrome

D. Hirschsprung’s disease 

E. Intussusception

Answer: Hirschsprung’s disease


Explanation

Hirschsprung’s disease is a congenital disorder characterized by the absence of ganglion cells in the distal portion of the colon, leading to functional obstruction. In the case described, the newborn presents on the fifth day of life with excessive abdominal distension, contouring of intestinal loops on the anterior abdominal wall, and passage of meconium after an enema. These features are indicative of intestinal obstruction.

The abdominal X-ray findings of dilation of the loops of the large intestine support the diagnosis of Hirschsprung’s disease. The absence of ganglion cells in the distal colon prevents normal peristalsis, leading to a functional obstruction. The meconium, which is normally passed shortly after birth, is retained in the affected segment of the colon, resulting in abdominal distension.

Let’s briefly review the other options and rule them out:

A. Portal hypertension: Portal hypertension is unlikely to cause the specific symptoms and X-ray findings described in the case. It is more associated with liver-related complications.

B. Stenosis of the ileum: Ileal stenosis might lead to obstruction but is less likely to present with the characteristic features of Hirschsprung’s disease, such as the absence of ganglion cells.

C. Ladd’s syndrome: Ladd’s syndrome is associated with malrotation of the intestines, but the clinical presentation and X-ray findings in this case are more consistent with Hirschsprung’s disease.

E. Intussusception: Intussusception usually presents with colicky abdominal pain, currant jelly-like stools, and a palpable abdominal mass, which are not described in the case.

In summary, the combination of symptoms, age of presentation, and X-ray findings strongly suggests Hirschsprung’s disease as the most likely diagnosis.

7. A 38-year-old patient complains of cramping abdominal pain, nausea, vomiti- ng, abdominal distension, inability to pass gas, constipation. The patient had a recent surgery for appendicitis in his medical hi- story. On palpation, the abdomen is painful in all its segments and tense. A splashing noi- se, unclear signs of peritoneal irritation, and hyperperistalsis are observed in the patient. What is the most likely diagnosis in this case? 

A. Diverticular colitis

B. Adhesive intestinal obstruction 

C. Diffuse peritonitis

D. Peptic ulcer disease of the stomach 

E. Colon cancer

Answer: Adhesive intestinal obstruction


Explanation

The clinical presentation of cramping abdominal pain, nausea, vomiting, abdominal distension, inability to pass gas, constipation, and a recent history of surgery for appendicitis is suggestive of adhesive intestinal obstruction. Adhesive intestinal obstruction occurs when bands of fibrous tissue (adhesions) form between abdominal organs or between an organ and the abdominal wall, leading to mechanical obstruction of the intestine.

The physical examination findings of a tense and painful abdomen in all segments, along with a splashing noise, unclear signs of peritoneal irritation, and hyperperistalsis, are consistent with a bowel obstruction. The splashing noise suggests the presence of fluid and gas within the obstructed intestine.

Let’s briefly review the other options and rule them out:

A. Diverticular colitis: Diverticular colitis is not a recognized medical term, and the symptoms described are not characteristic of diverticular disease.

C. Diffuse peritonitis: While peritonitis can cause abdominal pain and tenderness, the clinical picture of cramping abdominal pain, nausea, vomiting, and constipation is more indicative of an obstructive process rather than diffuse peritonitis.

D. Peptic ulcer disease of the stomach: Peptic ulcer disease typically presents with epigastric pain, not the diffuse abdominal pain and signs of obstruction described in this case.

E. Colon cancer: While colon cancer can cause bowel obstruction, the rapid onset of symptoms, recent surgery, and the presence of hyperperistalsis make adhesive intestinal obstruction a more likely diagnosis in this context.
In summary, considering the clinical presentation, recent surgery, and physical examination findings, adhesive intestinal obstruction is the most likely diagnosis in this case.

8. A 45-year-old woman complains of discomfort during reading, redness of the edges of her cichlids, and white foamy discharge in the corners of her palpebral fissures, observed for the past two months. Objectively, the following is observed: hyperemia and thickening of the loose eyelid margins and widened excretory ducts of the glands in the eyelid cartilage. What is the most likely diagnosis in this case?

A. Blennorrheal conjunctivitis 

B. Chronic canaliculitis 

C. Acute dacryoadenitis 

D. Adenoviral conjunctivitis 

E. Meibomian blepharitis

Answer: Meibomian blepharitis


Explanation

The patient’s symptoms of discomfort during reading, redness of the eyelid margins, and white foamy discharge in the corners of the palpebral fissures, along with the objective findings of hyperemia and thickening of the loose eyelid margins and widened excretory ducts of the glands in the eyelid cartilage, are indicative of Meibomian blepharitis.

Meibomian blepharitis is a common condition characterized by inflammation of the meibomian glands, which are responsible for producing the lipid component of the tear film. The widened excretory ducts and foamy discharge are consistent with the dysfunction of these glands.

Let’s briefly review the other options and rule them out:

A. Blennorrheal conjunctivitis: Blennorrheal conjunctivitis is typically caused by Neisseria gonorrhoeae or Chlamydia trachomatis and presents with purulent discharge. The symptoms and findings in this case are more suggestive of meibomian blepharitis.

B. Chronic canaliculitis: Canaliculitis involves inflammation of the tear ducts and is associated with tearing, not the foam-like discharge described in this case.

C. Acute dacryoadenitis: Acute dacryoadenitis involves inflammation of the lacrimal gland, and the clinical presentation is more likely to include pain, swelling, and redness in the upper outer portion of the eyelid.

D. Adenoviral conjunctivitis: Adenoviral conjunctivitis may cause redness, tearing, and a watery discharge, but the specific findings of meibomian gland involvement and foamy discharge are more characteristic of meibomian blepharitis.

In summary, considering the symptoms and objective findings, meibomian blepharitis is the most likely diagnosis in this case.

9. A 32-year-old woman complains of marked shortness of breath, dry cough, a fever of 39°C, and excessive sweating. Bacterioscopy of her sputum detected acid-fast bacteria [+] Mantoux test with 2 tuberculin units resulted in a papule 21 mm in size. X-ray visualizes numerous symmetrically located focal shadows 1-2 mm in size in both lungs.. The

shadows arc low-intensity and have blurry contours, What is the most likely diagnosis in

this case?

A. Chronic disseminated pulmonary tuberculosis

B. Focal tuberculosis

C. Caseous pneumonia

D. Sarcoidosis

E. Miliary pulmonary tuberculosis

Answer: Miliary pulmonary tuberculosis


Explanation

The clinical presentation of marked shortness of breath, dry cough, fever, and excessive sweating, along with the detection of acid-fast bacteria in sputum, a positive Mantoux test, and characteristic findings on chest X-ray (numerous symmetrically located low-intensity focal shadows 1-2 mm in size with blurry contours) strongly suggests miliary pulmonary tuberculosis.

Miliary tuberculosis occurs when Mycobacterium tuberculosis disseminates through the bloodstream, leading to the formation of tiny granulomas throughout the body, particularly in the lungs. The symptoms and chest X-ray findings described are indicative of widespread involvement of the lungs, with the small granulomas giving the appearance of miliary (millet seed-sized) lesions.

Let’s briefly review the other options and rule them out:

A. Chronic disseminated pulmonary tuberculosis: Miliary tuberculosis is a form of disseminated tuberculosis, and the clinical presentation and imaging findings are more characteristic of miliary pulmonary tuberculosis specifically.

B. Focal tuberculosis: The widespread, symmetrically located focal shadows described in both lungs are not consistent with focal tuberculosis.

C. Caseous pneumonia: The term “caseous pneumonia” is not commonly used, and the characteristic features described are more aligned with miliary tuberculosis.

D. Sarcoidosis: Sarcoidosis may present with pulmonary involvement, but the specific findings of acid-fast bacteria in sputum and a positive Mantoux test are not typical for sarcoidosis. The chest X-ray findings described are more suggestive of miliary tuberculosis.

In summary, considering the clinical presentation, microbiological findings, positive Mantoux test, and chest X-ray characteristics, the most likely diagnosis in this case is miliary pulmonary tuberculosis.

10. A 45-year-old patient complains of a skin rash on the trunk and upper and lower limbs. The patient associates the development of this rash with a recently experienced stressful situation. Objectively, there are inflammatory papules on the skin, which have a tendency to spread and are covered with loose silvery- yellow scales. When elements of the rash are scraped, the “stearin spot symptom” is observed. The patient’s genealogical history is normal. What is the most likely diagnosis in this case?

A. Psoriasis

B. Lichen ruber planus 

C. Eczema

D. Dermatophytosis 

E. Allergic dermatitis

Answer: Psoriasis


Explanation

The clinical presentation of inflammatory papules covered with loose silvery-yellow scales, a tendency to spread, and the “stearin spot symptom” (silvery scales easily come off, resembling stearin) is characteristic of psoriasis.

Psoriasis is a chronic inflammatory skin disorder that often presents with well-demarcated, raised, red papules covered with silvery scales. The tendency for lesions to spread and the presence of scales that easily come off during scraping are typical features. Stress is known to be a trigger for psoriasis exacerbations.

Let’s briefly review the other options and rule them out:

B. Lichen ruber planus: Lichen planus typically presents with flat-topped, shiny, violaceous papules, and it is not associated with the silvery scales seen in psoriasis.

C. Eczema: Eczema (atopic dermatitis) tends to present with red, inflamed, and pruritic skin, but the presence of loose silvery-yellow scales and the “stearin spot symptom” is more indicative of psoriasis.

D. Dermatophytosis: Dermatophytosis (fungal infection) usually presents with red, scaly lesions, but the specific characteristics mentioned in the case, such as loose silvery-yellow scales and the “stearin spot symptom,” are not typical for dermatophytosis.

E. Allergic dermatitis: Allergic dermatitis can present with various patterns of redness, inflammation, and itching, but it does not typically manifest with the specific features seen in psoriasis, such as silvery scales and the “stearin spot symptom.”

In summary, based on the clinical presentation and characteristic features, the most likely diagnosis in this case is psoriasis.

11. 11. A 42-year-old woman has been hospitalized with complaints of dull, aching pain in her lower back, more on the night side, and her body temperature sometimes increasing to subfebrile levels. The patient’s medical history states that 10 years ago during pregnancy she had a pain attack in her lumbar region on the right and a fever of 39°C. She underwent treatment with antibiotics. In recent years, she was feeling satisfactory. An increase in blood pressure has been observed for the past 5 years. Urinalysis revealed the following: protein – 0.66 g/L, leukocytes – 10-15 in sight, erythrocytes – 2-3 in sight. What is the most likely diagnosis in this case?

A. Renal urolithiasis

B. Chronic pyelonephritis

C. Chronic glomerulonephritis

D. Renal tuberculosis

E. Essential hypertension

Answer: Chronic pyelonephritis


Explanation

The patient’s symptoms of dull, aching pain in the lower back, particularly at night, along with subfebrile temperatures, a history of a pain attack during pregnancy, and the presence of proteinuria, leukocyturia, and hematuria in the urinalysis, are suggestive of chronic pyelonephritis.

Chronic pyelonephritis is a progressive inflammatory condition of the renal parenchyma and renal pelvis. The recurrent nature of the pain attacks, history of fever during pregnancy, and the presence of urinary abnormalities in the form of proteinuria, leukocyturia, and hematuria are indicative of chronic inflammation and scarring of the kidneys.

Let’s briefly review the other options and rule them out:

A. Renal urolithiasis: While renal stones can cause flank pain, the recurrent nature of the pain attacks, the history of fever during pregnancy, and the urinary abnormalities point more toward chronic pyelonephritis.

C. Chronic glomerulonephritis: Chronic glomerulonephritis may present with proteinuria and hematuria, but the characteristic features of dull, aching pain in the lower back and the recurrent nature of the symptoms are more suggestive of chronic pyelonephritis.

D. Renal tuberculosis: Renal tuberculosis can cause chronic inflammation of the kidneys, but the history of fever during pregnancy and the recurrent nature of the pain attacks are more consistent with chronic pyelonephritis.

E. Essential hypertension: While essential hypertension can cause an increase in blood pressure, it is not typically associated with the urinary abnormalities and recurrent pain attacks described in this case.

In summary, considering the clinical history, symptoms, and urinary findings, the most likely diagnosis in this case is chronic pyelonephritis.

12. Forensic autopsy of the body of a 59. year-old man, who died suddenly at home without signs of violent death, shows pink skin and mucosa, bright red blood, and bright red plethoric internal organs, Forensic toxicology blood testing detected 1.44 °) ethanol in his blood and carboxyhemoglobin levels of 55%. What substance has caused the poisoning that became the cause of death in this case?

A. Carbon monoxide

B. Aniline

C. Potassium cyanide

D. Arsenic

E. Alcohol

Answer: Carbon monoxide


Explanation

The clinical and forensic findings, including pink skin and mucosa, bright red blood, bright red plethoric internal organs, and elevated carboxyhemoglobin levels of 55%, are consistent with carbon monoxide poisoning.
Carbon monoxide (CO) is a colorless, odorless gas that is produced during incomplete combustion of carbon-containing fuels such as wood, gasoline, and natural gas. When inhaled, carbon monoxide binds to hemoglobin in the blood, forming carboxyhemoglobin. This reduces the oxygen-carrying capacity of the blood, leading to tissue hypoxia.

The elevated carboxyhemoglobin levels in the blood, along with the characteristic findings in the autopsy, are indicative of acute carbon monoxide poisoning. The presence of ethanol in the blood (1.44%) suggests co-ingestion of alcohol, but the primary cause of death is attributed to carbon monoxide.

Let’s briefly review the other options and rule them out:

B. Aniline: Aniline poisoning is associated with symptoms such as methemoglobinemia, but it does not typically produce the specific forensic findings described in the case.

C. Potassium cyanide: Cyanide poisoning usually leads to hypoxia, but the bright red coloration of internal organs and the carboxyhemoglobin levels are more indicative of carbon monoxide poisoning.

D. Arsenic: Arsenic poisoning may cause multi-organ toxicity, but the specific findings of bright red blood and plethoric internal organs are not characteristic of arsenic poisoning.

E. Alcohol: While alcohol (ethanol) was detected in the blood, the primary cause of death in this case is attributed to carbon monoxide poisoning. The elevated carboxyhemoglobin levels and forensic findings are more consistent with carbon monoxide as the toxic substance.

In summary, the most likely substance causing poisoning and leading to death in this case is carbon monoxide.

13. A 17-year-old patient objectively presents with no facial hair growth, gynecomastia, fat deposition on the hips, and a high-pitched voice. The patient is tall due to elongated lower limbs with a relatively short torso. Mental retardation is noted. Sex chromatin was detected in the buccal epithelium. What is the most likely diagnosis in this case? 

A. Patau syndrome 

B. Edwards syndrome 

C. Down syndrome 

E. Klinefelter syndrome 

D. Turner syndrome

Answer: Klinefelter syndrome 


Explanation

Klinefelter syndrome is a genetic disorder characterized by the presence of one or more extra X chromosomes in males (47,XXY). The typical karyotype is 47,XXY, where the affected individual has two X chromosomes and one Y chromosome. This condition results from non-disjunction during meiosis.

The clinical features described in the case, including no facial hair growth, gynecomastia (enlarged breasts), fat deposition on the hips, a high-pitched voice, tall stature due to elongated lower limbs and a relatively short torso, and mental retardation, are consistent with Klinefelter syndrome.

The detection of sex chromatin (Barr bodies) in the buccal epithelium is also a characteristic finding in individuals with Klinefelter syndrome. The presence of extra X chromosomes leads to incomplete masculinization and results in physical and cognitive manifestations.

Let’s briefly review the other options and rule them out:

A. Patau syndrome: Patau syndrome (Trisomy 13) is characterized by severe developmental abnormalities, including cleft lip and palate, polydactyly, and central nervous system malformations. It is not associated with the features described in the case.

B. Edwards syndrome: Edwards syndrome (Trisomy 18) presents with severe congenital abnormalities, including clenched fists, rocker-bottom feet, and cardiac defects. The features mentioned in the case are not typical of Edwards syndrome.

C. Down syndrome: Down syndrome (Trisomy 21) is characterized by distinct facial features, intellectual disability, and various physical abnormalities. The clinical presentation in this case is not consistent with Down syndrome.

D. Turner syndrome: Turner syndrome is characterized by a complete or partial absence of one X chromosome in females (45,X). The clinical features described, such as gynecomastia and elongated lower limbs, are not typical of Turner syndrome.

In summary, considering the physical and cognitive manifestations along with the karyotype findings, the most likely diagnosis in this case is Klinefelter syndrome.

14. A 38-year-old patient complains of a fever of 39°C, chills, profuse sweat, and dull lumbar pain that radiates into the suprapubic region Objectively, muscle tension is observed in the lumbar region, percussion test in the lumbar region provokes painful sensations on both sides. Complete blood count shows leukocytosis of 12 10/L. General urinalysis revealed the following: proteinuria -0.7 g/L leukocyturia 15-20 in sight, bacteriuria of over 100,000 bacteria per 1 mL of urine. What is the most likely diagnosis in this case? 

A. Acute glomerulonephritis 

B. Nephrolithiasis

C. Renal tuberculosis 

D. Acute pyelonephritis 

E. Acute cystitis

Answer: Acute pyelonephritis


Explanation

The patient’s presentation with fever, chills, profuse sweating, dull lumbar pain radiating into the suprapubic region, muscle tension in the lumbar region, and positive percussion test provoking painful sensations on both sides is highly suggestive of acute pyelonephritis.

Acute pyelonephritis is a bacterial infection of the kidneys, often caused by ascending urinary tract infections. The common symptoms include fever, chills, lumbar pain, and urinary symptoms. The percussion test, which provokes pain on both sides, is a classic finding associated with acute pyelonephritis.

The laboratory findings of leukocytosis (12 x 10^9/L), proteinuria (0.7 g/L), leukocyturia (15-20 in sight), and significant bacteriuria (over 100,000 bacteria per 1 mL of urine) are consistent with an active bacterial infection involving the urinary tract and kidneys.

Let’s briefly review the other options and rule them out:

A. Acute glomerulonephritis: Acute glomerulonephritis is characterized by hematuria, proteinuria, and oliguria. The presentation in this case, with a focus on fever, lumbar pain, and positive percussion test, is more indicative of acute pyelonephritis.

B. Nephrolithiasis: Nephrolithiasis (kidney stones) may cause similar symptoms of lumbar pain but typically lacks the systemic features of fever, chills, and leukocytosis seen in acute pyelonephritis.

C. Renal tuberculosis: Renal tuberculosis may present with persistent sterile pyuria, hematuria, and constitutional symptoms, but the bacterial infection with positive urine culture in this case points more toward acute pyelonephritis.

E. Acute cystitis: Acute cystitis is an infection of the bladder and presents with lower urinary tract symptoms (e.g., dysuria, urgency). The symptoms and laboratory findings described are more indicative of an upper urinary tract infection, such as acute pyelonephritis.

In summary, considering the clinical presentation and laboratory findings, the most likely diagnosis in this case is acute pyelonephritis.

15. A 32-year-old woman, gravida 1, at the term of 38 weeks complains of irregular spastic pain in her lower abdomen and lumbar region. The pain is observed for 2 days, the patient did not sleep well at night. External obstetric examination detected longitudinal lie, position 1, occiput anterior. increased tone of the uterus; the fetus in the fetal head is mobile and located above the entrance to the lesser pelvis. Vaginal exami- nation detects that the cervix is up to 2.5 cm, tilted backwards, unevenly softened, the external os is open to 0.5 cm, the internal os is closed. What is the most likely diagnosis in this case?

A. Pathological preliminary period 

B. Primary weakness of labor activity 

C. Precursors of childbirth

D. Secondary weakness of labor activity 

E. Discoordinated labor activity

Answer: Secondary weakness of labor activity


Explanation

The clinical presentation of irregular spastic pain in the lower abdomen and lumbar region, along with external obstetric examination findings of increased uterine tone, longitudinal lie, and occiput anterior position, suggests a form of dysfunctional labor. The fetus being mobile and located above the entrance to the lesser pelvis indicates that it has not descended into the birth canal.

Vaginal examination findings further support the diagnosis:

Cervix up to 2.5 cm: Cervical dilation is limited, indicating inadequate progress in the first stage of labor.Tilted backward cervix: This position may hinder the descent of the fetal head.Unevenly softened cervix: Inadequate cervical ripening can contribute to difficulty in labor progression.External os open to 0.5 cm, internal os closed:

Limited cervical dilation despite the onset of labor.Secondary weakness of labor activity refers to a situation where there is a significant delay in the progress of labor after the active phase has begun. It can be caused by various factors, including inadequate uterine contractions, ineffective maternal pushing efforts, and malposition or malpresentation of the fetus.

Let’s briefly review the other options and rule them out:A. Pathological preliminary period: The patient is at 38 weeks of gestation, indicating that she is in the term period rather than the preliminary period.B. Primary weakness of labor activity:

Primary weakness of labor activity refers to an initial failure of the uterine contractions to progress labor, which is not the case here as labor has already started.C. Precursors of childbirth: The clinical presentation and examination findings suggest a state of active labor rather than precursors of childbirth.

E. Discoordinated labor activity: Secondary weakness of labor activity can be considered a form of discoordination, but the specific findings and clinical context point more toward secondary weakness.
In summary, considering the clinical findings and stage of labor, the most likely diagnosis in this case is secondary weakness of labor activity.

16. A woman has been hospitalized into a gynecological inpatient department with complaints of pain in her lower abdomen and ‘dizziness. Her last menstruation was 6 weeks ago Obiectively her skin is pale, blood pressure-80/60 mmHg pulse 94/min. The sign of peritoneal ion in the lower segments is positive. Bimanual examination shows that the uterus is slightly engaged, the appendages are enlarged on the right and painful. Promtov’s sign is positive. What is the most likely diagnosis in this case?

A. Acute adnexitis 

B.Rupture cyst of the right ovary 

C. Interrupted ectopic pregnancy

D. Pedicle torsion of a cyst of the right ovary 

E. Ovarian apoplexy

Answer: Rupture cyst of the right ovary 


Explanation

The clinical presentation of pain in the lower abdomen, dizziness, pale skin, and signs of peritoneal irritation, along with blood pressure of 80/60 mmHg and a positive Promtov’s sign, suggests a sudden intra-abdominal hemorrhage. Ovarian apoplexy is a condition characterized by the rupture of a graafian follicle or a functional cyst of the ovary, leading to bleeding into the peritoneal cavity.
Key features supporting the diagnosis of ovarian apoplexy include:

Enlarged and painful appendages on the right during bimanual examination.Positive Promtov’s sign: Tenderness in the right iliac region during palpation.Blood pressure of 80/60 mmHg: Hypotension is a common finding due to blood loss.Rupture of an ovarian cyst may cause sudden, severe pain and internal bleeding. The released blood can irritate the peritoneum, resulting in signs of peritoneal irritation.

Let’s briefly review the other options and rule them out:A. Acute adnexitis: Acute inflammation of the fallopian tubes and ovaries would not typically present with the sudden onset of severe pain and hypotension seen in this case.B. Rupture cyst of the right ovary: This is a possible correct answer, as ovarian apoplexy involves the rupture of an ovarian cyst, leading to hemorrhage.C. Interrupted ectopic pregnancy:

Ectopic pregnancy can cause internal bleeding, but the bimanual examination findings and the absence of specific signs related to pregnancy make this less likely.D. Pedicle torsion of a cyst of the right ovary: Torsion of an ovarian cyst can lead to ischemia, but the acute presentation with hypotension is more suggestive of hemorrhage seen in ovarian apoplexy.

In summary, based on the clinical presentation and examination findings, the most likely diagnosis in this case is ovarian apoplexy, involving the rupture of a cyst on the right ovary.

17. A 10-month-old boy is poorly gaining weight. His mother complains about his constant persistent cough. Sputum is thick and viscous. The boy had pneumonia three times. His sweat chloride levels are over 80 mEq/L. What is the most likely diagnosis in this case? 

A. A foreign body in the bronchi

B. Mucoviscidosis (cystic fibrosis) 

C. Chronic bronchitis

D. Bronchial asthma.

E. Congenital lung abnormality

Answer: Mucoviscidosis (cystic fibrosis)


Explanation

The clinical presentation of a 10-month-old boy with poor weight gain, persistent cough, thick and viscous sputum, and a history of recurrent pneumonia is highly suggestive of cystic fibrosis (CF).
Key features supporting the diagnosis include:

Sweat chloride levels over 80 mEq/L: Elevated sweat chloride is a characteristic finding in cystic fibrosis. The sweat test is a reliable diagnostic tool for CF.Chronic respiratory symptoms: The persistent cough, thick sputum, and recurrent pneumonia are typical manifestations of CF, reflecting the impact of abnormal mucus production on the respiratory system.Poor weight gain:

Malabsorption due to pancreatic insufficiency is common in CF, leading to poor weight gain and failure to thrive.Let’s briefly review the other options and rule them out:A. A foreign body in the bronchi: While a foreign body can cause cough and respiratory symptoms, the recurrent nature of the symptoms and the elevated sweat chloride levels are more indicative of cystic fibrosis.

C. Chronic bronchitis: Chronic bronchitis in a 10-month-old child is less likely, and the characteristic thick and viscous sputum seen in CF is not a typical feature of chronic bronchitis.D. Bronchial asthma: Asthma can cause cough and respiratory symptoms, but the recurrent pneumonia, thick sputum, and elevated sweat chloride levels are not typical of asthma.

E. Congenital lung abnormality: Cystic fibrosis is a congenital condition that affects multiple organ systems, including the lungs. The specific clinical features described align more closely with CF than a generic congenital lung abnormality.

In summary, based on the clinical presentation and diagnostic findings, the most likely diagnosis in this case is cystic fibrosis (mucoviscidosis).

18. A 68-year-old woman has been hospi- talized with complaints of headache in the occipital region. marked dyspnea at rest that becomes worse when lying down, and dry cough. Objectively, the woman is in an orthopneic position, and acrocyanosis is observed. Over the lungs, weakened vesicular breathing can be heard with medium and fine vesicular wet crackles in the lower segments. The heart sounds are weakened, gallop rhythm is observed. Pulse – 102/min., blood pressure 210/110 mm Hg. The liver protrudes 2 cm from under the edge of the costal arch and is tender during palpation. There are edemas on the lower legs. What drugs should be used to provide emergency aid for this patient?

A. Coftriaxone, lazolvan (ambroxol) Intravenously

B. Nitroglycerin, furassmide intravenously

C. Labelmar intravenously, furosemide intramuscularly

D. Levofloxacin intravenously, ambroxol orally

E. Magnesium sulfate intravenously, furosemide intramuscularly

Answer: Nitroglycerin, furassmide intravenously


Explanation

The patient’s clinical presentation is suggestive of acute heart failure with pulmonary edema. Key features supporting this diagnosis include:
Headache, marked dyspnea, and orthopnea: Indicate respiratory distress associated with fluid accumulation in the lungs.Dry cough:

Can be a symptom of pulmonary congestion.Acrocyanosis: Suggestive of poor oxygenation and circulatory compromise.Weakened vesicular breathing with wet crackles: Indicates fluid in the alveoli, typical of pulmonary edema.Gallop rhythm and weakened heart sounds: Suggestive of heart failure.High blood pressure: The elevated blood pressure (210/110 mm Hg) may contribute to heart failure exacerbation.The appropriate emergency management for acute heart failure with pulmonary edema involves:

Nitroglycerin: A vasodilator that helps reduce preload and afterload on the heart, relieving symptoms of congestion.Furosemide: A loop diuretic that promotes diuresis and reduces fluid overload.Let’s briefly review the other options and rule them out:A. Ceftriaxone, ambroxol: These drugs are not the primary treatment for acute heart failure with pulmonary edema.

Ceftriaxone is an antibiotic, and ambroxol is a mucolytic.C. Labetalol, furosemide: Labetalol is a beta-blocker, and it may not be the ideal choice in a patient with acute heart failure. Furosemide is appropriate but should be administered intravenously for a faster effect.D. Levofloxacin, ambroxol: Levofloxacin is an antibiotic, and ambroxol is a mucolytic.

These drugs are not the primary treatment for acute heart failure.E. Magnesium sulfate, furosemide: While magnesium sulfate may have some utility in specific cases of heart failure, the primary medications for this condition are nitroglycerin and furosemide.

In summary, for a patient with acute heart failure and pulmonary edema, the most appropriate drugs for emergency aid are nitroglycerin and furosemide given intravenously.

19. A 35-year-old patient complains of weakness, excessive sweating, fatigability, pain in the right side during breathing, and a fever of 38°C. Objectively, the followi- ng is observed; respiratory rate 28/min., pulse 100/min. The right half of the chest lags behind during the act of breathing. Voi- ce tremor is not conducted on the right. Percussion detects a dull sound, breathing is weakened. The borders of the heart are shifted to the left. Complete blood count: leukocytes -12 10/L, band neutrophils-13%, lymphocytes 13%, ESR – 38 mm/hour. What is the most likely diagnosis in this case?

A. Pneumothorax

B. Right-sided pneumonia 

C. Exudative pleurisy 

D. Lung atelectasis

E. Infiltrative tuberculosis

Answer: Exudative pleurisy


Explanation

The patient’s clinical presentation, along with the physical examination and laboratory findings, suggests exudative pleurisy. Key features supporting this diagnosis include:
Weakness, excessive sweating, and fatigability: General symptoms that can be associated with systemic inflammation or infection.Pain in the right side during breathing and a fever of 38°C: Pleuritic chest pain and fever are common in pleurisy.

Respiratory rate of 28/min and pulse of 100/min: Indicate increased respiratory effort and heart rate, possibly due to pain and inflammation.Right half of the chest lags behind during breathing: This can be a sign of restricted movement of the right chest wall, which is characteristic of pleurisy.Voice tremor not conducted on the right: Suggests decreased transmission of sound through the lung on the right side.Dull percussion sound, weakened breathing, and shifted heart borders to the left: Indicate the presence of a pleural effusion on the right side.

Complete blood count showing leukocytosis with an elevated percentage of band neutrophils: Suggestive of an inflammatory or infectious process.Pleurisy is inflammation of the pleura, the membranes surrounding the lungs. Exudative pleurisy refers to the presence of an exudate (inflammatory fluid) in the pleural space, which can lead to symptoms such as pain, difficulty breathing, and decreased breath sounds on the affected side.
Let’s briefly review the other options and rule them out:

A. Pneumothorax: Pneumothorax typically presents with sudden-onset pleuritic chest pain, dyspnea, and decreased breath sounds on the affected side. The dull percussion sound and exudative pleurisy findings make pneumothorax less likely.B. Right-sided pneumonia: Pneumonia may cause similar symptoms, but the presence of a dull percussion sound and shifted heart borders suggests a pleural effusion rather than pneumonia alone.D. Lung atelectasis:

Atelectasis can present with decreased breath sounds and dullness on percussion, but the specific signs of pleurisy and the shift in heart borders are more indicative of exudative pleurisy.E. Infiltrative tuberculosis: While tuberculosis can involve the pleura, the presentation with unilateral pleuritic chest pain, dullness on percussion, and fever is more suggestive of exudative pleurisy.
In summary, considering the clinical presentation and findings, the most likely diagnosis in this case is exudative pleurisy.

20. During the last week a 26-year-old woman started doing many things that were new and unusual for her. In particular, she started painting the walls in the house entranceway on her own and was writing poems at night, whi- le making illustrations for them. Objectively, she is talkative, quickly changes topics of conversation, actively gestures, jokes, and flirts with men. She believes that she could have been a great actress, writer, and artist, and invites everyone to attend her evening “art recitals”. What psychopathological condition is observed in the patient?

A. Hebephrenic syndrome 

B. Hysterical neurosis syndrome 

C. Manic syndrome

D. Pseudoparalytic dementia 

E. Excited catatonia

Answer: Manic syndrome


Explanation

The patient’s presentation is indicative of a manic episode, characterized by an elevated, expansive, or irritable mood, increased energy, and excessive involvement in activities. Key features supporting this diagnosis include:
Engaging in new and unusual activities: Individuals in a manic episode often exhibit impulsive and goal-directed behavior.Painting walls, writing poems, making illustrations:

These activities are examples of increased goal-directed activity and creativity, which are typical in a manic episode.Talkative, quick changes of topics, active gestures, joking, and flirting: These behaviors are consistent with the increased energy, sociability, and impulsivity seen in mania.Grandiosity: The belief that she could have been a great actress, writer, and artist reflects grandiose delusions often present in manic episodes.Inviting everyone to attend her “art recitals”: Increased sociability and a desire for attention are common in manic episodes.

Let’s briefly review the other options and rule them out:A. Hebephrenic syndrome: Hebephrenic schizophrenia is characterized by disorganized thinking, bizarre behavior, and inappropriate affect. The patient’s presentation is more consistent with the elevated mood and increased activity seen in mania.B. Hysterical neurosis syndrome:

Hysterical neurosis (now termed conversion disorder) is characterized by the presence of neurological symptoms without a clear medical explanation. The symptoms in the case are more indicative of a mood disorder (mania).D. Pseudoparalytic dementia: Pseudoparalytic dementia is not a recognized psychiatric diagnosis.

The symptoms described are more in line with a manic episode.E. Excited catatonia: Excited catatonia may involve increased motor activity, but it is often associated with other features of catatonia, such as posturing or grimacing. The symptoms in this case are more indicative of a manic episode.

In summary, considering the elevated mood, increased energy, impulsivity, and other features, the most appropriate psychopathological condition in this case is manic syndrome.

21. A patient complains of a rash and itchi- ng that becomes worse in the evening and at night and has been observed for 2 weeks already. Objectively, a papulovesicular rash with its elements arranged in pairs is observed on the lateral areas of the chest and abdomen and in the interdigital folds. What is the most likely diagnosis in this case?

A. Scabies

B. Toxicoderma

C. Psoriasis

D. Eczema

E. Neurodermatitis

Answer: Scabies


Explanation

The patient’s complaint of a rash and itching, particularly worsening in the evening and at night, along with the observed papulovesicular rash with elements arranged in pairs on the lateral areas of the chest and abdomen and in the interdigital folds, is indicative of scabies.

Key features supporting the diagnosis of scabies include:

Itching that worsens in the evening and at night: This is a characteristic symptom of scabies and is associated with increased mite activity during these times.Papulovesicular rash arranged in pairs: The classic appearance of scabies lesions includes small papules, vesicles, or burrows arranged in linear or S-shaped patterns, often seen between the fingers, on the wrists, and in other intertriginous areas.

Let’s briefly review the other options and rule them out:B. Toxicoderma: Toxicoderma refers to a rash caused by exposure to various toxins or drugs. The description of the rash in pairs is not typical of toxicoderma.C. Psoriasis: Psoriasis typically presents with well-defined erythematous plaques with silvery scales.

The distribution and appearance described are not consistent with psoriasis.D. Eczema: Eczema (atopic dermatitis) may cause itching and a rash, but the specific distribution and arrangement in pairs are not typical for eczema.E. Neurodermatitis: Neurodermatitis refers to a skin condition characterized by chronic itching and scratching, often resulting in localized areas of thickened skin.

The distribution and arrangement described are not typical for neurodermatitis.

In summary, based on the clinical presentation and the characteristic features of the rash, the most likely diagnosis in this case is scabies.

22. A premature baby born at 34 weeks of gestation presents with the following at 4 hours after birth: tachypnea, respiration with a seesaw motion, sternum depression, expiratory murmurs. Respiratory rate- 80/min.  Auscultation detects weakened breathing with non-constant heterogeneous crackles over the lungs. X-ray of the lungs shows air bronchogram and a nodose-reticular lung pattern. What is the most likely diagnosis in this case?

A. Pulmonary atelectasis

B. Massive meconium aspiration syndrome 

C. Hyaline membrane disease 

D. Birth injury

E. Neonatal pneumonia

Answer: Hyaline membrane disease


Explanation

The clinical presentation and diagnostic findings are characteristic of hyaline membrane disease (HMD), also known as respiratory distress syndrome (RDS), which is common in premature infants.
Key features supporting the diagnosis of HMD include:

Premature baby born at 34 weeks of gestation: Prematurity is a major risk factor for HMD.Tachypnea, respiration with a seesaw motion, sternum depression, expiratory murmurs: These are signs of respiratory distress, which is a hallmark of HMD.Respiratory rate of 80/min: Tachypnea is a common feature in infants with HMD.Auscultation detects weakened breathing with non-constant heterogeneous crackles over the lungs:

Heterogeneous crackles indicate the presence of airway obstruction and fluid accumulation in the alveoli, consistent with HMD.X-ray of the lungs shows air bronchogram and a nodose-reticular lung pattern: Classic radiographic findings in HMD include a ground-glass appearance, air bronchograms, and a reticular-nodular pattern.

Let’s briefly review the other options and rule them out:A. Pulmonary atelectasis: While atelectasis can occur in premature infants, the specific findings, such as air bronchogram and the nodose-reticular pattern on X-ray, are more indicative of HMD.B. Massive meconium aspiration syndrome:

Meconium aspiration syndrome may cause respiratory distress in newborns, but the X-ray findings described are not typical for meconium aspiration.D. Birth injury: The presentation and X-ray findings are more suggestive of a respiratory condition (HMD) rather than a birth injury.

E. Neonatal pneumonia: While neonatal pneumonia can present with respiratory distress, the classic X-ray findings in this case are more consistent with HMD.
In summary, based on the premature birth, clinical presentation, and characteristic X-ray findings, the most likely diagnosis in this case is hyaline membrane disease (respiratory distress syndrome)

23. Examination of a 4-day-old baby detected vesicles with seropurulent content on the neck, back of the head, and buttocks. The patient’s condition is satisfactory, the child is active, all newborn reflexes can be fully induced, the umbilical cord is at the stage of mummification, the umbilical region is normal. What is the most likely diagnosis in this case?

A. Phlegmon

B. Epidermolysis bullosa 

C. Neonatal pemphigus 

D. Miliaria

E. Vesiculo Pustulosis

Answer: E. Vesiculo Pustulosis


Explanation

The clinical presentation of vesicles with seropurulent content on the neck, back of the head, and buttocks in a 4-day-old baby, along with a satisfactory condition, normal newborn reflexes, and a mummified umbilical cord, is suggestive of vesiculopustular disorders, commonly known as benign neonatal pustulosis (BNP).

Key features supporting the diagnosis of vesiculo-pustulosis (BNP) include:

Vesicles with seropurulent content: The presence of vesicles or pustules with clear or purulent content is characteristic of this benign and self-limiting condition.Satisfactory condition and normal newborn reflexes: Typically, infants with BNP are otherwise healthy and exhibit normal behavior and reflexes.Mummified umbilical cord:

The normal appearance of the umbilical cord and surrounding area suggests that the skin findings are not related to umbilical issues.Let’s briefly review the other options and rule them out:A. Phlegmon: Phlegmon refers to a spreading, diffuse inflammation, often involving the subcutaneous tissues. The vesicular and pustular lesions described are not consistent with phlegmon.B. Epidermolysis bullosa:

Epidermolysis bullosa is a genetic disorder characterized by blistering of the skin in response to minor trauma. However, the presentation in this case is more likely related to a transient, benign condition.C. Neonatal pemphigus: Neonatal pemphigus is a rare autoimmune blistering disorder that can be present at birth.

However, the clinical presentation and the overall health of the infant make BNP a more likely diagnosis.D. Miliaria: Miliaria, or heat rash, typically presents as small red papules rather than vesicles with seropurulent content.
In summary, based on the clinical presentation and the nature of the skin lesions, the most likely diagnosis in this case is benign neonatal pustulosis (vesiculo-pustulosis).

24. Assess the physical development of a 10- year-old girl according to the regression scale, if her body weight and chest circumference parameters are within ± 1 sigma.

A. Harmonious

B. Low

C. Disharmonious 

D. High

E. Average

Answer: Harmonious


Explanation

The use of the term “sigma” in this context refers to standard deviations (σ) from the mean in a normal distribution. When the parameters (body weight and chest circumference) are within ± 1 sigma, it means they fall within one standard deviation above or below the mean.

In the context of physical development assessment:

“Harmonious” suggests that the various aspects of physical development (such as weight and chest circumference) are proportionate and in balance with each other.”Low” or “Disharmonious” might indicate that one or more aspects of physical development are below the average or not in proportion with others.

“High” might indicate that one or more aspects of physical development are above the average or not in proportion with others.”Average” would suggest that the parameters fall within the average range for the given age.Given that both body weight and chest circumference are within ± 1 sigma, the most appropriate categorization for the physical development of the 10-year-old girl would be “Harmonious.”

25. A patient came to a doctor with complaints of enlarged cervical and axillary lymph nodes and heaviness in the left hypochondrium. During palpation, the lymph nodes are soft, painless, and not fused together. Splenomegaly is observed. Complete blood count: leukocytes-70-10/L, lymphocytes- 80%, Gumprecht shadows. What is the most likely diagnosis in this case?

A. Burkitt lymphoma 

B. Lymphogranulomatosis 

C. Non-Hodgkin lymphoma 

D.Chronic lymphocytic leukemia 

E. Acute leukemia

Answer: Chronic lymphocytic leukemia


Explanation

The clinical presentation of enlarged cervical and axillary lymph nodes, heaviness in the left hypochondrium, and splenomegaly, along with soft, painless, and non-fused lymph nodes, is characteristic of chronic lymphocytic leukemia (CLL). Key features supporting the diagnosis include:

Leukocytosis with lymphocytosis: A complete blood count (CBC) revealing a high white blood cell count (leukocytosis) with a predominance of lymphocytes is a characteristic finding in CLL.Gumprecht shadows: Gumprecht shadows, also known as smudge cells, are fragile lymphocytes that rupture during blood smear preparation, resulting in irregularly shaped nuclei.

This finding is typical in CLL.Soft, painless lymph nodes: Unlike some aggressive lymphomas, lymph nodes in CLL are often soft, painless, and not fused together.Let’s briefly review the other options and rule them out:A. Burkitt lymphoma: Burkitt lymphoma typically presents with rapidly growing, often painless lymphadenopathy, and it is associated with the Epstein-Barr virus (EBV).

The clinical features and chronic course in this case make CLL a more likely diagnosis.B. Lymphogranulomatosis: This term is often used to refer to Hodgkin lymphoma. The presentation described, with predominantly soft, painless lymph nodes, is more suggestive of CLL.C. Non-Hodgkin lymphoma:

Non-Hodgkin lymphoma can have diverse presentations, but the chronic course, lymphocytosis, and characteristic Gumprecht shadows are more consistent with CLL.E. Acute leukemia: Acute leukemias typically present with symptoms of bone marrow failure, such as anemia, thrombocytopenia, and neutropenia. The chronic nature of the symptoms and the predominant lymphocytosis make CLL a more appropriate diagnosis.
In summary, based on the clinical presentation, laboratory findings, and characteristic features, the most likely diagnosis in this case is chronic lymphocytic leukemia (CLL).

26 A 50-year-old man has been hospitalized in a severe condition. ECG revealed signs of acute myocardial infarction. Objectively, the patient is anxious, tense, disoriented in the place and time, and correctly oriented in his own person. The patient experiences auditory and bright visual hallucinations of a frightening nature, under the influence of which he is agitated and prone to aggressive actions. The patient expresses fragmentary delusions. What leading psychopathological syndrome is observed in the patient?

A. Hallucinosis

B. Delirious syndrome 

C. Amentive syndrome

 D. Paranoid syndrome 

E. Oneiroid syndrome

Answer: Oneiroid syndrome


Explanation

The patient’s presentation includes signs of acute myocardial infarction along with prominent psychiatric symptoms. Key features supporting the diagnosis of oneiroid syndrome include:
Anxiety, tension, and disorientation in place and time:

These symptoms are characteristic of an altered mental state, commonly seen in oneiroid syndrome.Correct orientation in person: Despite disorientation in place and time, the patient is correctly oriented in terms of self, which is a characteristic feature of oneiroid states.Auditory and bright visual hallucinations of a frightening nature:

Hallucinations, especially vivid and frightening, are often part of oneiroid syndromes.Agitation and aggressive actions: Oneiroid states can lead to heightened emotional states, including agitation and potentially aggressive behavior.Fragmentary delusions: The presence of fragmentary delusions further supports the diagnosis of a oneiroid state.Oneiroid syndrome is a term used to describe a state of clouded consciousness, marked by confusion, hallucinations, and emotional turmoil. It is often associated with acute stress, severe medical conditions, or psychiatric disorders.

Let’s briefly review the other options and rule them out:A. Hallucinosis: While hallucinations are present, the term “hallucinosis” typically refers to a more chronic and persistent condition.B. Delirious syndrome: Delirium involves an acute and fluctuating disturbance of consciousness with reduced ability to focus, sustain, or shift attention.

The term “oneiroid syndrome” is more specific to the altered state described.C. Amentive syndrome: Amentia refers to a state of mental confusion or impairment. However, the term “oneiroid syndrome” better captures the specific features described.D. Paranoid syndrome: Paranoid syndrome involves a preoccupation with delusions or prominent hallucinations of a persecutory nature.

The patient’s presentation includes elements beyond paranoia.E. Oneiroid syndrome: This term is the most fitting to describe the patient’s altered mental state, including hallucinations, confusion, and emotional turmoil.
In summary, based on the symptoms described, the leading psychopathological syndrome observed in the patient is oneiroid syndrome.

27. A 32-year-old primipara woman developed intense contractions that lasted 55-60 seconds with an interval of 1-2 minutes between them. Objectively, the disengagement of the fetal head begins. The perineum protrudes excessively and has a height of 4 cm. The skin of the perineum is pale and tense. After a contraction stopped, a thin stream of blood appeared from the genital opening. Specify the further tactics of managing the delivery.

A. Application of obstetrical forceps 

B. Episiotomy

C. Vacuum extraction of the fetus 

D. Waiting tactics

E. Cesarean section

Answer: Application of obstetrical forceps


Explanation

The described situation involves a 32-year-old primipara woman with intense contractions, disengagement of the fetal head, excessive protrusion of the perineum, pale and tense perineal skin, and the appearance of a thin stream of blood from the genital opening. These signs suggest that the second stage of labor is progressing, but there may be difficulties or a delay in the descent of the fetal head.

Obstetrical forceps are instruments designed to assist in the delivery of the fetal head during the second stage of labor. They can be applied to the fetal head to facilitate its rotation and descent through the birth canal. Forceps are typically used when there is a need for expedited delivery due to concerns about maternal or fetal well-being.

Key indications for the application of obstetrical forceps may include:

Prolonged second stage of labor: If the second stage of labor is prolonged, and there is a risk of maternal exhaustion or fetal distress, forceps can be used to assist in a timely delivery.

Failure to progress: If there is a failure of the fetal head to descend despite adequate maternal efforts, forceps may be applied to aid in the descent and rotation.

Maternal fatigue: In cases where the mother is exhausted and unable to continue pushing effectively, forceps can be used to facilitate delivery.
Fetal distress: If there are signs of fetal distress and a need for prompt delivery, forceps application can be considered.

It’s important to note that the decision to use forceps should be made by a skilled healthcare provider based on a thorough assessment of the specific clinical situation. While forceps can be a valuable tool for assisted vaginal delivery, their use is not without risks, and the decision should prioritize the safety and well-being of both the mother and the baby.

In summary, if there are indications of a delay in the descent of the fetal head and concerns about maternal or fetal well-being, the application of obstetrical forceps may be considered to assist in the delivery process.

28. A 14-year-old girl complains of vaginal bleeding that lasts for 10 days and occurred after a three-month delay of menstruation. She had menarche at the age of 13, her menstrual cycle is irregular. Complete blood count detects hemoglobin levels of 90 g/L. What is the most likely diagnosis in this case?

A. Juvenile uterine bleeding

B. Uterine malformation 

C. Werlhof’s disease

D. Hormone-producing uterine tumor 

E. Cervical polyp

Answer: Juvenile uterine bleeding


Explanation

Juvenile uterine bleeding, also known as dysfunctional uterine bleeding (DUB) or anovulatory bleeding, is a condition characterized by abnormal uterine bleeding in the absence of an underlying structural or systemic cause. This condition often occurs in adolescent girls and young women and is associated with irregular menstrual cycles.

Key points supporting the diagnosis of juvenile uterine bleeding in this case include:

Age and Menstrual History: The patient is 14 years old, which is within the age range when menstrual irregularities are common during the establishment of regular cycles. She had menarche at the age of 13, and her menstrual cycles are described as irregular.

Vaginal Bleeding: The complaint of vaginal bleeding lasting for 10 days after a three-month delay of menstruation is consistent with the pattern seen in anovulatory bleeding.

Complete Blood Count (CBC): The CBC reveals hemoglobin levels of 90 g/L, indicating anemia. Prolonged or heavy menstrual bleeding can lead to iron-deficiency anemia.

Differential diagnoses for abnormal uterine bleeding in adolescents may include hormonal imbalances, uterine malformations, bleeding disorders (Werlhof’s disease), hormone-producing tumors, and cervical polyps.

However, given the patient’s age, irregular menstrual cycles, and the absence of other apparent causes, juvenile uterine bleeding is the most likely diagnosis.

It’s important for the patient to undergo a thorough evaluation by a healthcare provider to confirm the diagnosis, rule out other potential causes, and determine an appropriate management plan. Management may involve addressing anemia, regulating menstrual cycles, and providing education on menstrual hygiene and contraception if needed. The patient needs to undergo.

29. A 29-year-old patient who suffers from hormone-dependent Fluorography detects a round shadow of bronchial asthma. medium intensity with clear even contours in C2 of the right lung. Around the shadow, there are several polymorphic focal shadows. There is a calcination at the root of the lung Examination detects a banbox resonance in the percussion sound over the lungs, diffuse dry crackles can be heard. Blood test detects no changes. Mantoux test reaction with 2 tuberculin units PPD-L resulted in a papule 22 mm in size. What is the most likely diagnosis in this case?

A. Eosinophilic infiltration 

B. Aspergilloma 

C. Peripheral cancer 

D. Tuberculoma 

E. Pneumonia

Answer: Tuberculoma


Explanation

Several key findings point toward the diagnosis of tuberculoma:

Hormone-dependent Fluorography: This suggests a history of tuberculosis or a condition that is influenced by hormonal changes.

Round Shadow on Fluorography: The presence of a round shadow with clear even contours in C2 of the right lung is a characteristic radiological finding associated with tuberculoma. Tuberculomas are nodular lesions that can develop in the lungs as a result of tuberculosis infection.

Polymorphic Focal Shadows: The additional polymorphic focal shadows around the main shadow are consistent with the granulomatous nature of tuberculomas, which can manifest as multiple nodules.

Calcification at the Root of the Lung: The presence of calcification at the lung’s root further supports the possibility of a chronic granulomatous process, which is common in tuberculosis.

Percussion Sound and Crackles: The dull percussion sound and diffuse dry crackles suggest pathology within the lung tissue, which aligns with the granulomatous nature of tuberculomas.

Mantoux Test Reaction: A positive Mantoux test reaction (22 mm papule) indicates exposure to Mycobacterium tuberculosis. Tuberculomas can result from the reactivation of latent tuberculosis.

While other conditions such as peripheral cancer, pneumonia, eosinophilic infiltration, and aspergilloma may present with lung shadows, the combination of the mentioned findings, including the characteristic round shadow, calcification, and positive Mantoux test, strongly suggests tuberculoma in this case.

It’s important for the patient to undergo further diagnostic tests, such as imaging studies (e.g., chest CT scan) and possibly a biopsy, to confirm the diagnosis and determine the appropriate management, which may involve anti-tuberculosis medications.The patient needs to undergo

30. A 35-year-old man complains of cough and shortness of breath during physical exertion. This health condition has been observed for the past 4 years. The patient has been working at a foundry for the last 14 years, where the concentration of quartz dust in the air of the workshop was 4 times higher than the maximum permissible concentration. Auscultation detected weakened respiration. Chest X-ray revealed emphysema and fine macular shadows in all lung fields. What is the most likely diagnosis in this case? 

A. Silicosis

B. Pulmonary tuberculosis

C. Asbestosis

D. COPD

E. Chronic bronchitis

Answer:  Silicosis


Explanation

Occupational Exposure: The patient has a history of working at a foundry for the last 14 years, where the concentration of quartz dust in the air was significantly higher than the maximum permissible concentration. Silicosis is an occupational lung disease caused by the inhalation of silica dust, commonly found in jobs involving mining, construction, or foundry work.

Chronic Respiratory Symptoms: The patient presents with a chronic cough and shortness of breath during physical exertion, which are characteristic symptoms of silicosis. The onset of symptoms after several years of exposure is consistent with the slow progression of the disease.

Weakened Respiration and Chest X-ray Findings: Auscultation reveals weakened respiration, and chest X-ray shows emphysema and fine macular shadows in all lung fields. Silicosis can lead to the development of nodular densities, often described as fine macular shadows, along with emphysema, contributing to the respiratory symptoms.

Long Latency Period: Silicosis typically has a long latency period, and symptoms may not manifest until several years after exposure to high levels of silica dust.

While other respiratory conditions such as pulmonary tuberculosis, asbestosis, COPD, and chronic bronchitis may present with similar symptoms, the occupational history and characteristic radiological findings strongly suggest silicosis as the most likely diagnosis.

It’s essential for the patient to undergo further evaluation, including pulmonary function tests and possibly a high-resolution CT scan, to assess the severity of lung involvement and determine the appropriate management, which may include supportive measures and cessation of further exposure to silica dust.

31. A man complains of dizziness and vomiting. Vomitus is dark-colored. The patient’s history states that he often drinks alcohol Esophagogastroduodenoscopy detected that the contents of the stomach resembled “coffee grounds”, in the area of the cardia, there were four longitudinal fissures in the mucosa, from which a small amount of blood was leaking. What is the most likely diagnosis

in this case?

A. Gastric cardia ulcer

B. Bleeding from gastric varices 

C.Mallory-Weiss syndrome

D. Zollinger-Ellison syndrome 

E. Erosive gastritis

Answer: Mallory-Weiss syndrome


Explanation

Clinical Presentation: The patient presents with symptoms of dizziness and vomiting, and the vomitus is dark-colored, indicative of the presence of blood. This suggests upper gastrointestinal bleeding.
Alcohol Consumption: The patient’s history mentions frequent alcohol consumption. Excessive alcohol intake is a risk factor for Mallory-Weiss syndrome.

Esophagogastroduodenoscopy (EGD) Findings: The EGD reveals that the contents of the stomach resemble “coffee grounds,” which is a sign of partially digested blood. Additionally, four longitudinal fissures are observed in the mucosa near the cardia, from which a small amount of blood is leaking. These findings are consistent with Mallory-Weiss syndrome.

Mallory-Weiss Syndrome: This syndrome is characterized by longitudinal mucosal lacerations at the gastroesophageal junction, often associated with severe vomiting or retching. It can lead to upper gastrointestinal bleeding.

While other conditions such as gastric cardia ulcer, bleeding from gastric varices, Zollinger-Ellison syndrome, and erosive gastritis may also cause upper gastrointestinal bleeding, the specific findings of longitudinal fissures near the cardia are typical of Mallory-Weiss syndrome.

Management may involve supportive measures, addressing the underlying cause (e.g., alcohol cessation), and, in severe cases, interventions to stop the bleeding, such as endoscopic therapy.
It’s important for the patient to receive prompt medical attention and further evaluation to determine the extent of the bleeding and appropriate management.

32. During the medical examination of a patient, the doctor noted a precorneal injection of the vessels in the sclera of the eyes (a “violet-blue corolla”), conjunctivitis, cheilosis, angular stomatitis, hypertrophic glossitis. What vitamin is deficient in this case, causing such clinical presentation in the patient?

A. Pyridoxine 

B. Thiamine 

C. Riboflavin 

D. Niacin

E. Cyanocobalamin

Answer: Riboflavin 


Explanation

The clinical presentation described, including precorneal injection of vessels in the sclera, conjunctivitis, cheilosis, angular stomatitis, and hypertrophic glossitis, is characteristic of a deficiency in riboflavin, also known as vitamin B2.

Riboflavin plays a crucial role in various metabolic pathways, including those involved in energy production. The deficiency of riboflavin can manifest with a range of symptoms, and the combination of ocular signs (precorneal injection, conjunctivitis) along with oral manifestations (cheilosis, angular stomatitis, hypertrophic glossitis) is indicative of riboflavin deficiency.

It’s important to note the specific clinical signs associated with each vitamin deficiency. In this case, the “violet-blue corolla” refers to the precorneal injection of vessels in the sclera, a distinctive sign seen in riboflavin deficiency.

Therefore, option C, Riboflavin, is the correct answer based on the clinical presentation and the known functions of the vitamin.

33. A 32-year-old patient complains of pain in the left leg, observed for a year, with intermittent limping after walking 50-100 meters and periodical nocturnal pain. Objectively. The left leg is pale and cold. Arterial pulsation is absent in the left lower leg. What is the most likely diagnosis in this case?

A.Obliterating endarteritis 

B. Aortic thrombosis

C. Raynaud syndrome

D. Leriche syndrome 

E. Aortic embolism

Answer: Obliterating endarteritis


Explanation

The clinical presentation described, including pain in the left leg after walking, intermittent limping, pale and cold left leg, and absent arterial pulsation in the left lower leg, is indicative of a vascular disorder affecting the lower extremities.

Obliterating endarteritis, also known as thromboangiitis obliterans or Buerger’s disease, is characterized by inflammation and thrombosis of small and medium-sized arteries and veins, primarily in the extremities. The symptoms often include claudication (limping or pain during walking), and in severe cases, it can lead to critical limb ischemia with pain at rest, cold extremities, and absent pulses.

In this case, the absence of arterial pulsation in the left lower leg and the presence of intermittent limping after walking are suggestive of a vascular occlusive disease, and obliterating endarteritis fits this clinical scenario.
Therefore, option A, Obliterating endarteritis, is the most likely diagnosis based on the patient’s symptoms and physical findings.

34. A 37-year-old patient complains of general weakness, spastic pain in the lower segments of the abdomen, mainly in the left iliac region, and loose stools with mucus and blood up to 18 times a day. The disease onset was acute and occurred three days ago with chills, feeling hot, and a headache. The patient’s general condition is moderately severe, body temperature of 37.8°C. Palpati- on detects spastic and painful sigmoid colon. What is the most likely diagnosis in this case? 

A. Amoebiasis

B. Non-specific ulcerative colitis 

C. Shigellosis

D. Salmonellosis

E. Yersiniosis

Answer: Shigellosis


Explanation

The patient’s symptoms of general weakness, spastic pain in the lower abdomen (especially in the left iliac region), and loose stools with mucus and blood up to 18 times a day are indicative of an acute bacterial infection affecting the gastrointestinal tract.

Shigellosis is caused by Shigella bacteria and typically presents with symptoms such as abdominal pain, diarrhea with blood and mucus, and fever. The spastic and painful sigmoid colon on palpation is consistent with the involvement of the colon, which is a common site of infection in shigellosis.

The acute onset of symptoms with chills, feeling hot, and a headache further supports the diagnosis of an acute infectious process.

Therefore, based on the clinical presentation and symptoms, option C, Shigellosis, is the most likely diagnosis in this case.

35. A 29-year-old woman complains of infertility and an oligomenorrhea type of abnormal menstrual cycle. Objectively, she has a height of 160 cm and a body weight of 91 kg, growth of hair is observed on her face and thighs. Bimanual examination detects enlarged dense ovaries 5×6 cm in size on both sides. These data were confirmed by ultrasound. What is the most likely diagnosis in this case?

A.Sclerocystic ovarian syndrome (Stein- Leventhal)

B. Premenstrual syndrome 

C. Ovarian androblastoma 

D. Adrenogenital syndrome 

E. Chronic bilateral adnexitis

Answer: Sclerocystic ovarian syndrome (Stein- Leventhal)


Explanation

The patient’s presentation, including infertility, oligomenorrhea, hirsutism (growth of hair on face and thighs), and bimanual examination findings of enlarged dense ovaries, is indicative of polycystic ovary syndrome (PCOS), also known as sclerocystic ovarian syndrome or Stein-Leventhal syndrome.

PCOS is a common endocrine disorder among women of reproductive age. The characteristic features include menstrual irregularities (oligomenorrhea or amenorrhea), hyperandrogenism (hirsutism, acne), and polycystic ovaries on imaging studies like ultrasound.

In this case, the combination of infertility, oligomenorrhea, hirsutism, and enlarged dense ovaries seen on ultrasound supports the diagnosis of PCOS.

Therefore, option A, Sclerocystic ovarian syndrome (Stein-Leventhal), is the most likely diagnosis based on the patient’s clinical presentation and examination findings.

36 A 27-year-old woman complains of sharp pain in her lower abdomen and dizziness. Her last menstruation was 2 weeks ago. HCG test results are negative. Objectively, her skin is pale, blood pressure 80/60 mm Hg, pulse 92/min. The abdomen is tense, painful more on the right in its lower segments. Vagi-nal examination detects a normal-sized uterus, the appendages are painful to palpation, the posterior fornix overhangs. What is the most likely diagnosis in this case?

A. Pedicle torsion of an ovarian cyst

B. Exacerbation of chronic right-sided adnexitis

C. Acute appendicitis 

D. Ovarian apoplexy 

E. Ectopic pregnancy

Answer: Pedicle torsion of an ovarian cyst


Explanation

Clinical Presentation: The patient presents with sharp pain in her lower abdomen, dizziness, and a history of a missed period. The negative human chorionic gonadotropin (HCG) test indicates that she is not pregnant.
Hypotension and Pallor: The patient has hypotension (blood pressure 80/60 mm Hg) and pallor. These findings are suggestive of hypovolemic shock, which can be associated with intra-abdominal bleeding.
Tense and Painful Abdomen:

The abdomen is described as tense and more painful on the right in its lower segments. Abdominal pain associated with tension can be indicative of a surgical emergency.
Vaginal Examination Findings: Vaginal examination reveals a normal-sized uterus, painful appendages, and the posterior fornix overhanging. These findings are consistent with a possible adnexal (ovarian and fallopian tube) pathology.

Likely Diagnosis: The combination of severe lower abdominal pain, hypotension, and pallor in a woman of reproductive age, along with the findings on vaginal examination, is highly suggestive of pedicle torsion of an ovarian cyst. Torsion can compromise blood flow to the ovary and lead to ischemia, pain, and potentially rupture.

While other conditions such as exacerbation of chronic adnexitis, acute appendicitis, ovarian apoplexy, and ectopic pregnancy may cause abdominal pain, the specific combination of symptoms, examination findings, and the absence of pregnancy make pedicle torsion of an ovarian cyst the most likely diagnosis.

Emergency surgical intervention may be required to address the torsion and prevent further complications. It’s essential for the patient to receive prompt medical attention and undergo imaging studies, such as ultrasound, to confirm the diagnosis and guide management.The patient needs to receive

37. During physical exercises, a 32-year-old patient suddenly felt lack of air, weakness, chest pain on the right that radiated into the right shoulder, shortness of breath, and palpitations. Objectively, the patient’s condition is severe, tachycardia is up to 100/min., blood pressure 90/60 mm Hg, respiratory rate-28/min., the right half of the chest lags behind during breathing. Percussion detects tympanic sound on the right, no respiratory sounds were detected there. The body temperature is normal. What is the most likely diagnosis in this case?

A. Pneumonia

B. Pulmonary infarction 

C. Vascular collapse 

D. Myocardial infarction 

E. Spontaneous pneumothorax

Answer: Spontaneous pneumothorax


Explanation

Sudden Onset of Symptoms: The patient experienced a sudden onset of symptoms during physical exercises, including lack of air, weakness, chest pain on the right radiating into the right shoulder, shortness of breath, and palpitations.

Severe Condition and Tachycardia: The patient’s condition is described as severe, and there is tachycardia (heart rate up to 100/min), which can be a response to decreased oxygen levels.
Blood Pressure and Respiratory Rate: Blood pressure is 90/60 mm Hg, indicating possible hemodynamic compromise.

The respiratory rate is elevated (28/min), reflecting increased respiratory effort.

Asymmetry in Chest Movement: The right half of the chest lags behind during breathing, suggesting impaired expansion of the right lung.

Percussion and Auscultation Findings: Percussion detects a tympanic sound on the right, and no respiratory sounds are heard on that side. These findings are consistent with decreased air entry into the right lung.
Normal Body Temperature: The absence of fever makes conditions like pneumonia less likely.

Likely Diagnosis: The combination of sudden-onset symptoms, asymmetry in chest movement, percussion findings, and absence of respiratory sounds on the right side is characteristic of spontaneous pneumothorax. In this condition, air enters the pleural space, leading to lung collapse and presenting with acute respiratory distress and chest pain.

While other conditions like pulmonary infarction, vascular collapse, myocardial infarction, and pneumonia may cause some similar symptoms, the specific findings in this case, along with the absence of risk factors for pulmonary embolism or myocardial infarction, make spontaneous pneumothorax the most likely diagnosis.

Emergency intervention, such as tube thoracostomy, may be required to relieve the pneumothorax and re-expand the lung. Prompt medical attention is crucial in managing this condition.

38. A 30-year-old woman on the fifth day after the physiological childbirth complained of swelling of her left breast, pain, reddened skin, and a fever of 38°C. Objectively, her left mammary gland is enlarged, the skin there is red, with cyanosis in the upper-outer quadrant. A sharply painful infiltrate without clear borders can be palpated. Expressing miIk does not bring relief. What is the most likely diagnosis in this case?

A. Erysipelas

B. Infected breast cyst

C. Lactostasis

D. Breast cancer

E. Acute infiltrative mastitis

Answer: Acute infiltrative mastitis


Explanation

Postpartum Period: The patient is in the fifth day after physiological childbirth, which is a vulnerable period for the development of breast-related issues.

Symptoms: The complaint of swelling of the left breast, pain, reddened skin, and fever (38°C) indicates an inflammatory process.
Objective Findings:

Enlarged left mammary gland.Red skin, especially with cyanosis in the upper-outer quadrant.Sharply painful infiltrate without clear borders.Pain not relieved by expressing milk.Clinical Presentation: The combination of symptoms and objective findings is suggestive of an acute inflammatory condition involving the breast tissue.

Likely Diagnosis: Acute infiltrative mastitis is the most likely diagnosis. This condition often occurs in lactating women and is characterized by an inflammatory process within the breast tissue. The pain, swelling, redness, and fever are indicative of an infection.

Differential Diagnosis:

Erysipelas is a skin infection, but the involvement of the mammary gland and the presence of an infiltrate suggest a deeper process.An infected breast cyst would typically present as a localized issue rather than involving the entire breast.

Lactostasis refers to milk stasis, but the presence of pain, redness, and fever may indicate an infectious process.Breast cancer is less likely to present acutely after childbirth and is usually not associated with skin changes and fever.Management: Treatment typically involves antibiotics to address the infection, and supportive measures such as warm compresses and pain management.

Given the postpartum period, the inflammatory signs involving the entire breast, and the absence of relief with milk expression, acute infiltrative mastitis is the most appropriate diagnosis, and prompt medical attention is essential.

39. A 5-year-old child complains of attacks of spasmodic cough. The child has been ill for 2 weeks after the dry cough appeared. After the treatment that was ineffective, relapses started occurring. During a cough attack, the child’s face becomes red and neck veins swell. Objectively, the patient is pale, with edematous face and hemorrhages in the sclera. Auscultation detects a bandbox resonance over the lungs on percussion and dry crackles. X-ray detects increased transparency of the lung fields and intensified bronchial patterns. Blood test results are as follows: leukocytes-16-10″/L, lymphocytes 72%, ESR -4 mm/hour. What is the most likely diagnosis in this case?

A. A foreign body in the airways 

B. Tuberculous bronchadenitis 

C. Laryngotracheitis

D. Pertussis

E. Adenovirus infection

Answer: Pertussis


Explanation

Clinical Presentation:
Attacks of spasmodic cough lasting for 2 weeks after the onset of dry cough suggest a prolonged coughing phase, characteristic of pertussis.Facial congestion (red face) and neck vein swelling during cough attacks are known as paroxysms and are classic features of pertussis.Objective Findings:

Pale appearance with an edematous face and hemorrhages in the sclera can result from the intensity of coughing spells.Auscultation findings of bandbox resonance on percussion and dry crackles are consistent with pertussis-related lung manifestations.X-ray findings of increased transparency of lung fields and intensified bronchial patterns are indicative of pertussis-associated changes.Blood Test Results:

Leukocytosis (elevated leukocyte count) with a predominant lymphocytosis (72%) is a common finding in pertussis.Normal ESR (erythrocyte sedimentation rate) suggests an inflammatory process without significant elevation.Differential Diagnosis:

A foreign body in the airways may present with coughing but is less likely to cause the prolonged paroxysmal cough seen in pertussis.Tuberculous bronchadenitis would typically have different radiographic findings and a more chronic course.Laryngotracheitis may cause coughing, but the characteristic paroxysms are more suggestive of pertussis.Adenovirus infection may present with respiratory symptoms, but the specific coughing pattern and characteristic findings on X-ray make pertussis more likely.Likely Diagnosis:

Pertussis, also known as whooping cough, is a highly contagious respiratory tract infection caused by Bordetella pertussis. The characteristic paroxysms of coughing, the whooping sound, and the prolonged course align with the clinical presentation.Management:

Treatment often involves supportive care and antibiotics in the early stages of the illness.Vaccination is a crucial preventive measure against pertussis.Given the age of the child, the prolonged coughing episodes, and the classic features of pertussis, it is the most likely diagnosis. Prompt medical attention and appropriate management are essential.

40. A 54-year-old woman complains of a fogged vision in her right eye, rainbow circles in her vision field, headache, and nausea. Wi- thin the last month she twice experienced a similar condition, but back then all the signs eventually disappeared and her sight was restored. Currently, all the signs have been persisting for over 2 days. Objectively, the patient has eyelid edema, congestive injection of the eyeball, corneal opacity, shallow anterior chamber of the eye, and dilated pupil that is unresponsive to the light. Her intraocular pressure is 48 mm Hg. What is the most likely diagnosis in this case?

A. Iritis

B. Prolonged acute attack of glaucoma 

C. Keratitis

D. Iridocyclitis

E. Cyclitis

Answer: Prolonged acute attack of glaucoma


Explanation

Clinical Presentation:
The patient complains of fogged vision, rainbow circles (halos) in the vision field, headache, and nausea.These symptoms are indicative of increased intraocular pressure, which is characteristic of a glaucoma attack.Duration and Persistence:

The fact that the signs have been persisting for over 2 days suggests a prolonged nature of the acute attack.Objective Findings:

Eyelid edema, congestive injection of the eyeball, corneal opacity, shallow anterior chamber of the eye, and a dilated pupil that is unresponsive to light are consistent with acute glaucoma.An intraocular pressure of 48 mm Hg is significantly elevated and supports the diagnosis.Differential Diagnosis:

Iritis (A), Keratitis (C), Iridocyclitis (D), and Cyclitis (E):These conditions involve inflammation of different parts of the eye and may present with various symptoms, but the combination of symptoms and signs described aligns more closely with glaucoma.Glaucoma is characterized by elevated intraocular pressure, which can lead to optic nerve damage and vision loss if not promptly treated.Likely Diagnosis:

Prolonged acute attack of glaucoma refers to a sustained increase in intraocular pressure, leading to symptoms and signs of glaucoma that persist over an extended period.The symptoms and objective findings in this case are consistent with an ongoing glaucoma attack.Management:

Urgent medical intervention is required to reduce intraocular pressure and prevent further damage.Treatment may involve medications (e.g., topical or systemic medications to lower intraocular pressure) and, in some cases, surgical procedures.Given the persistent symptoms, signs, and elevated intraocular pressure, the diagnosis of a prolonged acute attack of glaucoma is the most likely scenario, requiring immediate attention and intervention.

41. A neonatologist examines a full-term baby born from the second pregnancy, second full-term delivery with the body weight of 3980 & During the delivery primary weakness of the labor activity was observed and obstetric assistance was used. Objectively, the right hand is adducted to the trunk and rotated, there are no movements in the shoulder and elbow joints, the “doll’s arm”sign is observed, the hand is in the posi- tion of palmar flexion. The baby breastfeeds, suckles actively. What is the most likely di- agnosis in this case?

A. Humerus fracture on the right 

B. Duchenne-Erb paresis

C. Dislocation of the right shoulder 

D. Total brachial plexus paresis 

E. Dejerine-Klumpke paresis

Answer: Duchenne-Erb paresis


Explanation

Clinical Presentation:
The baby presents with weakness and lack of movement in the right arm.The “doll’s arm” sign is observed, indicating decreased muscle tone and movement in the affected limb.The hand is in the position of palmar flexion.Birth History:

The description of primary weakness of labor activity and obstetric assistance during delivery suggests a traumatic event during childbirth.Duchenne-Erb Paresis:
Duchenne-Erb paresis, also known as Erb’s palsy, is a condition that results from injury to the upper brachial plexus during childbirth.It commonly occurs when there is excessive pulling or stretching of the baby’s neck during delivery.Clinical Signs in Duchenne-Erb Paresis:

Weakness and lack of movement in the affected arm.”Doll’s arm” sign: The affected arm hangs limply, resembling a doll’s arm.Position of palmar flexion: The hand may be flexed at the wrist and fingers.Functionality:
Despite the weakness, the baby can breastfeed and suckle actively, indicating that the injury primarily affects the motor function rather than sensory or reflex functions.Differential Diagnosis:

Other conditions such as fractures (A), dislocations (C), total brachial plexus paresis (D), or Dejerine-Klumpke paresis (E) may present with different clinical features.In Duchenne-Erb paresis, the characteristic presentation involves the upper brachial plexus.Management:

Management may include physical therapy and rehabilitation to promote the development of motor function in the affected arm.Prognosis varies, and some cases may require surgical intervention.Given the clinical presentation and the context of childbirth-related trauma, Duchenne-Erb paresis is the most likely diagnosis in this case.

42. A routine examination of a 22-year-old woman at week 30 of her pregnancy detected isolated bacteriuria twice in her urine. The pregnancy course is without pathology. Speci- fy the further tactics of managing this pregnant woman.

A. Physiotherapy

B. Dynamic monitoring

C. Prescription of ampicillin 

D. Prescription of ciprofloxacin 

E. Phytotherapy

Answer: Prescription of ampicillin 


Explanation

Isolated Bacteriuria in Pregnancy:
Isolated bacteriuria in pregnancy refers to the presence of bacteria in the urine without clinical symptoms of urinary tract infection (asymptomatic bacteriuria).Pregnant women are at an increased risk of urinary tract infections due to physiological changes in the urinary tract.Significance:

Asymptomatic bacteriuria is associated with an increased risk of developing symptomatic urinary tract infections, which can lead to adverse outcomes such as preterm birth and low birth weight.Guidelines:
According to medical guidelines, pregnant women with asymptomatic bacteriuria are typically treated with antibiotics to prevent the progression to a urinary tract infection.Choice of Antibiotics:

Ampicillin is considered safe for use in pregnancy and is often chosen for the treatment of asymptomatic bacteriuria.Ciprofloxacin is generally avoided during pregnancy due to concerns about potential adverse effects on fetal development.Prevention of Complications:

Treating asymptomatic bacteriuria helps prevent complications and ensures a healthier outcome for both the mother and the baby.Monitoring:
Dynamic monitoring (Option B) is not sufficient in the case of bacteriuria. Active treatment is recommended to prevent complications.Alternative Approaches:

Phytotherapy (Option E) may not be the primary choice for treating bacteriuria. Antibiotics are the standard approach to eliminate bacteria.In summary, prescribing ampicillin is a common and appropriate management strategy to address asymptomatic bacteriuria in pregnant women, helping to reduce the risk of complications during pregnancy.

43. Examination of pork detected 2 trichinella in 24 sections on the compressorium. What should be done with this meat?

A. Used in public catering networks 

B. Used to make sausages 

C.Technically utilized

D. Deep frozen

E. Cut into small pieces and boiled

Answer: Technically utilized


Explanation

Trichinella:
Trichinella is a parasitic nematode that can infect humans and animals, especially pigs.Trichinosis is the disease caused by the ingestion of raw or undercooked meat containing Trichinella larvae.Detection in Pork:
Finding Trichinella in pork during examination indicates a potential risk of trichinosis if the meat is consumed.Options:

Used in public catering networks (Option A): This is not advisable, as it poses a risk of spreading the infection to consumers.Used to make sausages (Option B): Sausages made from infected meat can also transmit the infection.Technically utilized (Option C): This is the correct option.

Technically utilized means that the meat should be disposed of in a way that prevents it from being consumed or entering the food supply chain.Deep frozen (Option D): Freezing may kill the parasites, but it is not a foolproof method, and there are regulations regarding the freezing process for this purpose.Cut into small pieces and boiled (Option E): Boiling may kill the parasites, but it is not considered a reliable method for eliminating Trichinella.Public Health Measures:

Technically utilizing the meat is a public health measure to prevent the spread of trichinosis to consumers.In summary, when Trichinella is detected in pork, the appropriate action is to technically utilize the meat, ensuring that it is safely disposed of to prevent the risk of infection to humans.

44. A 12-year-old girl complains of weakness, dizziness, headache, and a fever of 38°C. Objectively, her body temperature is 378°C. her mucosa and skin are pale, her pharynx is without changes. Palpation detects submandibular and cervical lymph nodes that are enlarged to 2 cm, dense and painless. No pathological changes of internal organs were detected. Complete blood count: erythrocytes 2.8 1012/1, hemoglobin 85 g/L, color index 0.9, leukocytes 10-10/1, eosinophils – 0%, band neutrophils-1%, segmented neutrophils lymphocytes – 47%, reticulocytes – 0.5%, – 8%. platelets-60-10/L, blast cells – 44%. What is the most likely diagnosis in this case? 

A. Chronic lymphocytic leukemia 

B. Infectious mononucleosis 

C. Acute erythromyclosis 

D. Acute leukemia

E. Lymphogranulomatosis

Answer: Acute leukemia


Explanation

Clinical Presentation:
The patient presents with weakness, dizziness, headache, and a fever of 38°C. These symptoms are indicative of a systemic illness.Physical Examination Findings:

Enlarged submandibular and cervical lymph nodes (2 cm, dense, painless) suggest lymphadenopathy, a common finding in hematological malignancies.Complete Blood Count (CBC):

Erythrocytes 2.8 x 10^12/1, hemoglobin 85 g/L, color index 0.9: Indicates anemia, which is common in leukemia due to bone marrow involvement.Leukocytes 10-10/1: Leukocytosis, a characteristic feature of leukemia.Eosinophils 0%, band neutrophils 1%, segmented neutrophils 47%: Abnormal distribution of leukocytes.Platelets 60 x 10^9/L: Thrombocytopenia, another common finding in leukemia.Blast cells 44%: Presence of blast cells (immature cells) in the peripheral blood is a hallmark of acute leukemia.Diagnosis:

The clinical presentation, CBC findings, and the high percentage of blast cells in the blood are consistent with acute leukemia.Chronic lymphocytic leukemia (Option A) is less likely in this case, as it typically occurs in older adults and has a more indolent course.Infectious mononucleosis (Option B) may present with lymphadenopathy and atypical lymphocytes, but the high percentage of blast cells suggests a hematological malignancy rather than a viral infection.Acute erythromyelosis (Option C) is not a recognized medical condition. It may be a term used incorrectly.Next Steps:

Further diagnostic tests, including bone marrow aspiration and biopsy, immunophenotyping, and cytogenetic analysis, would be needed to confirm the specific type of acute leukemia (e.g., acute lymphoblastic leukemia or acute myeloid leukemia) and guide treatment.In summary, the patient’s symptoms, physical examination findings, and CBC results strongly suggest acute leukemia, and further diagnostic tests would be necessary for confirmation and to determine the specific subtype.

45. A woman complains of severe pain in her throat on the left, difficulty swallowing and mouth opening, elevated body temperature, and generally feeling unwell. She was provisionally diagnosed with acute pharyngitis. Examination detects a trismus of the masticatory muscles, the left tonsil is displaced towards the midline, the anterior palatal arch is infiltrated and protruding. The regional lymph nodes on the left are enlarged and painful to palpation. What is the most likely diagnosis in this case?

A. Infectious mononucleosis 

B. Lacunar tonsillitis 

C. Scarlet fever

D. Peritonsillar abscess 

E. Tonsillar tumor

Answer: Peritonsillar abscess


Explanation

Clinical Presentation:
Severe pain in the throat, difficulty swallowing, and trismus (difficulty opening the mouth) suggest a deeper infection involving the peritonsillar area.Physical Examination Findings:

Left tonsil displaced towards the midline: This can occur due to the accumulation of pus in the peritonsillar space, causing displacement.Infiltrated and protruding anterior palatal arch: Inflammation and swelling in the peritonsillar region.Enlarged and painful regional lymph nodes on the left: Lymphadenopathy is common in response to infection.Diagnosis:

The clinical presentation is consistent with a peritonsillar abscess, which is a localized collection of pus in the peritonsillar space.Differential Diagnosis:

Infectious mononucleosis (Option A): While it can cause sore throat and lymphadenopathy, trismus and displacement of the tonsil are not typical features.Lacunar tonsillitis (Option B): In lacunar tonsillitis, there is inflammation within the tonsillar crypts, but it typically doesn’t lead to the formation of a distinct abscess.Scarlet fever (Option C): Causes a rash and sore throat but doesn’t typically result in a peritonsillar abscess.Tonsillar tumor (Option E): Unlikely given the acute onset and characteristic signs of infection.Management:

Peritonsillar abscess often requires drainage of the pus, which can be done through needle aspiration or incision and drainage.Complications:

If left untreated, a peritonsillar abscess can lead to serious complications such as airway obstruction and the spread of infection.In summary, the clinical features, including severe throat pain, trismus, and a displaced tonsil, are indicative of a peritonsillar abscess. Confirmatory tests and appropriate management, such as drainage, would be necessary to address the condition.

46. A 19-year-old girl complains of weakness, moderate headache, and sore throat when swallowing. She has been ill for 3 days already Objectively, her body temperature is 38.2°C. Generalized lymphadenopathy signs of tonsillopharyngitis and moderate hepatosplenomegaly are observed. A complete blood count detected 35% of atypical mononuclear cells. What study shoud she be referred for, to verify the diagnosis?

A. Blood testing for IgManti-HHV-7 

B. Blood testing for IgManti-VCA EBV 

C. Blood testing for IgManti-HHV-6 

D. Blood testing for IgManti-CMV 

E. Blood testing for total antibodies to HIV

Answer: Blood testing for IgManti-VCA EBV 


Explanation

Clinical Presentation:
Symptoms such as weakness, sore throat, and headache along with generalized lymphadenopathy and hepatosplenomegaly suggest a viral infection.Atypical Mononuclear Cells:
The presence of 35% atypical mononuclear cells in the complete blood count is a key finding.Likely Diagnosis:

Infectious mononucleosis (IM), commonly caused by the Epstein-Barr virus (EBV), is a leading consideration.Confirmatory Test:

Testing for antibodies specific to EBV can help confirm the diagnosis.IgM anti-VCA (Viral Capsid Antigen): The presence of IgM antibodies against VCA is indicative of recent EBV infection.Differential Diagnoses:

Option A (IgM anti-HHV-7): Human Herpesvirus 7 is less commonly associated with mononucleosis-like symptoms compared to EBV.Option C (IgM anti-HHV-6): Human Herpesvirus 6 is also less commonly associated with mononucleosis.Option D (IgM anti-CMV): Cytomegalovirus (CMV) can cause a mononucleosis-like syndrome, but the symptoms and serological findings are typically different.Option E (Total antibodies to HIV): HIV infection can present with similar symptoms, but the serological profile is different. An HIV test may be considered separately.Management:

Supportive care, as infectious mononucleosis is a self-limiting condition.Caution with the use of antibiotics (e.g., amoxicillin) due to the risk of developing a rash.In conclusion, given the clinical presentation and the presence of atypical mononuclear cells, testing for IgM anti-VCA EBV antibodies is the most appropriate to confirm the diagnosis of infectious mononucleosis caused by EBV.

47. A 57-year-old man complains of a pain in his chest, dyspnea during physical exertion, excessive sweating, constant subfebrile body temperature, and cough that produces blood-streaked sputum. He has been smoking for approximately 40 years (2 packs a day) and says that he frequently has “pneumonias”. Survey chest X-ray shows a triangular shadow in the middle lobe of the right lung. One of the apices of the shadow points towards the lung root. Cardiac and mediastinal shadows are displaced towards the affected area. What is the most likely diagnosis in this case?

A. Pneumoconiosis

B. Chronic bronchitis

C. Right-sided pleuropneumonia 

D. Tuberculosis of the right lung

E. Cancer of the right lung

Answer: Cancer of the right lung


Explanation

Clinical Presentation:
Chest pain, dyspnea, excessive sweating, subfebrile body temperature, and hemoptysis are concerning symptoms.Long-term smoking history (40 years, 2 packs a day) is a major risk factor for lung cancer.Chest X-ray Findings:

A triangular shadow in the middle lobe of the right lung with one apex pointing towards the lung root is a classic description of a mass lesion, possibly a tumor.Displacement of cardiac and mediastinal shadows towards the affected area is indicative of a space-occupying lesion.Diagnostic Considerations:

Option A (Pneumoconiosis): Usually associated with occupational exposure to dust or mineral particles, and the presentation is not consistent with the described symptoms.Option B (Chronic bronchitis): While smoking is a risk factor, the described findings and chest X-ray are more suggestive of a mass lesion.Option C (Right-sided pleuropneumonia): Unlikely given the chronic nature of symptoms, especially with the long smoking history.

Option D (Tuberculosis of the right lung): Tuberculosis can cause hemoptysis, but the chest X-ray findings are not typical for pulmonary tuberculosis.Option E (Cancer of the right lung): The most likely diagnosis given the clinical presentation, chest X-ray findings, and smoking history.Further Evaluation:

Confirmatory tests, such as CT scans and biopsy, would be necessary to confirm the diagnosis of lung cancer.Management:

Referral to oncology for further evaluation and consideration of treatment options, which may include surgery, radiation, or chemotherapy.In conclusion, the clinical presentation, imaging findings, and the patient’s smoking history strongly suggest lung cancer as the most likely diagnosis. Further confirmatory tests and oncology consultation are warranted.

48. What food product can cause dj- phyllobotriasis, if insufficiently processed?”

A. Beef

B. Poultry 

C.Fish 

D. Mutton 

E. Pork

Answer: Fish 


Explanation

Diphyllobothriasis: It is a parasitic infection caused by tapeworms of the genus Diphyllobothrium. The most common species involved is Diphyllobothrium latum. This tapeworm is known as the fish tapeworm.
Mode of Transmission:

Consumption of raw or undercooked fish, particularly freshwater fish, is the primary mode of transmission of Diphyllobothriasis.Life Cycle:

Humans become infected by ingesting larvae present in raw or undercooked fish.Once ingested, the larvae develop into adult tapeworms in the small intestine.Clinical Presentation:

Infections may be asymptomatic.Symptoms, when present, can include abdominal discomfort, diarrhea, and weight loss.Prevention:

Thoroughly cooking or freezing fish can kill the larvae and prevent infection.Educating individuals about the risks of consuming raw or undercooked fish is essential.Options Analysis:

Option A (Beef): Not associated with Diphyllobothriasis; it is more related to tapeworms like Taenia saginata.Option B (Poultry):

Poultry is not a common source of Diphyllobothriasis.Option C (Fish): Correct. Fish, especially freshwater fish, is a well-known source of Diphyllobothriasis.Option D (Mutton): Mutton is not commonly associated with Diphyllobothriasis.

Option E (Pork): Pork is not a typical source of Diphyllobothriasis.In conclusion, option C (Fish) is the correct answer, as insufficiently processed or undercooked fish is a common source of Diphyllobothriasis.

49. A 35-year-old woman complains of Frequent, painful, difficult urination, constant urges to urinate, and several drops of blood appearing in her urine at the end of the process. She fell ill suddenly, after overexposure to cold, when the complaints above appeared. Body temperature – 36.6°C. During palpation, pain is observed in the area of the urinary bladder. Ultrasound shows a small amount of urine in the bladder, the walls of the bladder are edematous and uniformly thickened. Urinalysis detects leukocyturia (30-40 in sight), proteinuria (0.099 g/L), and erythrocytosis (5-7 unchanged erythrocytes). What is the most likely diagnosis in this case?

A. Ureterocele

B. Bladder tumor

C. Acute salpingo-oophoritis 

D. Bladder concrement 

E. Acute cystitis

Answer: Acute cystitis


Explanation

Clinical Presentation:
Frequent, painful, and difficult urination are characteristic symptoms of cystitis.Constant urges to urinate are common.Hematuria (blood in the urine) is often observed, which explains the drops of blood appearing in the urine.Onset After Cold Exposure:

The sudden onset of symptoms after overexposure to cold may suggest that the cold exposure triggered the development of cystitis.Physical Examination:
Pain during palpation in the area of the urinary bladder is indicative of inflammation.Ultrasound Findings:

The small amount of urine in the bladder, along with edematous and uniformly thickened bladder walls, is consistent with cystitis.Urinalysis:

Leukocyturia (elevated white blood cells), proteinuria, and erythrocytosis (presence of red blood cells) are typical findings in acute cystitis.Differential Diagnosis:
Option A (Ureterocele): Unlikely; ureterocele is a congenital abnormality of the ureter and is not typically associated with sudden onset symptoms after cold exposure.Option B (Bladder Tumor): Less likely given the acute onset of symptoms and the characteristic findings in acute cystitis.

Option C (Acute Salpingo-oophoritis): Involves inflammation of the fallopian tubes and ovaries, not directly related to the urinary symptoms described.Option D (Bladder Concrement): Unlikely; a concrement (stone) might cause symptoms, but it doesn’t correlate with the sudden onset after cold exposure.Therefore, the most likely diagnosis in this case is acute cystitis (Option E).

50. A 17-year-old girl complains of pain and swelling of the second finger on the right hand. She had a manicure done three days ago. The pain appeared on the second day. Objectively, the periungual ridge is edematous, hyperemic overhangs the nail plate, painful during palpation. What is the most likely diagnosis in this case? 

A. Subungual felon 

B. Subcutaneous felon 

C. Erysipeloid

D. Cutaneous felon 

E. Paronychia

Answer: Paronychia


Explanation

Clinical Presentation:
Pain and swelling of the second finger after a recent manicure suggest an infection around the nail.Physical Examination Findings:

Periungual ridge edema, hyperemia, and pain during palpation are typical signs of inflammation in the paronychial tissues.Diagnosis:

Paronychia (Option E): It is an infection of the tissues adjacent to the nail, often caused by bacteria (commonly Staphylococcus or Streptococcus) or fungi. This condition can result from trauma or manipulation of the cuticle or nail folds during activities like manicure.Differential Diagnosis:

Option A (Subungual Felon): Refers to an infection beneath the nail, not around the nail folds.Option B (Subcutaneous Felon): Involves deeper tissue infections in the fingertip, not typically associated with nail manipulation.Option C (Erysipeloid):

Presents as a skin infection caused by Erysipelothrix rhusiopathiae, commonly seen in individuals handling fish or meat; not likely in the context of a recent manicure.Option D (Cutaneous Felon): Refers to a superficial infection of the fingertip pad; does not involve the nail folds.Therefore, based on the symptoms and the recent manicure, the most likely diagnosis is paronychia (Option E).
Join the conversation
0% Complete